renal1 Flashcards Preview

CVPR > renal1 > Flashcards

Flashcards in renal1 Deck (360)
Loading flashcards...
1
Q

Nephron

A

the nephron consists of blood supply and an epithelial tube, known as the tubule. The blood supply is remarkable, since it consists of two capillary beds in series; these are known as the glomerular and peritubular capillaries, respectively.

2
Q

Glomerular Filtration

A

The first thing the nephron does to is filter the plasma into the initial part of the tubule. This process of filtration allows the free passage of water and solutes into the tubule, but retains larger colloids (i.e. proteins, lipid aggregates, etc.) and circulating blood cells in the blood.

3
Q

Tubular Reabsorption

A

Once in the tubule, the kidney can recapture the filtered components that it wishes to regulate by a process generically known as reabsorption, which involves the transport of substances across the epithelia cell layer, and it involves highly selective transporters. Thus as the plasma filtrate flows through the tubule, the kidney can selectively regulate the rate of reabsorption of individual ECF components; these rates of reabsorption are constantly varied so that just enough ECF components are returned to the circulating plasma to achieve ECF constancy.

4
Q

Excretion

A

Regulated substances in excess of those required to maintain ECF balance simply pass through the tubule and are excreted as part of the urinary output.

5
Q

Regulation by tubular secretion

A

The forgoing description strictly applies to the two most abundant components of the ECF, water and sodium. However, other substances undergo a regulated process of secretion as part of their tubular handling to achieve ECF balance. Secretion is an epithelial transport process that involves the movement of substances from the blood or blood side of the tubule into the tubular lumen. It also often involves specific molecular transporters (channels and/or pumps). It’s also useful to mention that some substances can undergo both reabsorption and secretion within the tubule. As we shall see, the regulation of potassium (K+) involves just such a situation.

6
Q

Non-ECF functions of the renal system

A

The kidneys perform a number of other functions not directly related to their role in ECF homeostasis. For example, the kidneys produce erythropoetin (EPO) from its precursor synthesized in the liver. Thus anemia has to be considered as a potential problem in the management of renal failure cases. In addition, the kidneys can contribute to gluconeogenesis, or the production of glucose from other metabolites in the situation of fasting or starvation.

7
Q

The renin-angiotensin axis

A

However, the kidneys do play a hormonal role in the regulation of blood pressure by regulating the circulating levels of the potent vasopressor hormone angiotensin.

8
Q

The Renin-Angiotensin axis

A

In this pathway, the primary regulatory event occurs when a decrease in blood pressure is sensed by baroreceptors, causing the kidneys to increase their secretion of the renal enzyme renin. The purpose of this enzyme is to cleave the 13 amino acid peptide prohormone angiotensingen to the10 amino acid angiotensin I (AgI), which is biologically inactive. However AgI circulates to the lungs, where it is cleaved by converting enzyme into the active form, angiotensin II (AgII). This hormone then circulates, causing arteriolar smooth muscle to constrict; the increased peripheral resistance thus causes a rise in blood pressure (MAP) back towards normal. The main thing to remember here is that the level of renin is rate-limiting for the production of AgII and thus determines the status of the axis. It’s useful to note that a number of antihypertensive agents are directed at this system, particularly the angiotensin converting enzyme (A.C.E.) inhibitors and angiotensin receptor antagonists.

9
Q

Structure of the filtration apparatus

A

Filtration takes place across the capillary loops into Bowman’s capsule of the tubule; here the filtrate starts its journey through the tubule where it is processed into urine. The arterioles on either side of the glomerular capillary bed serve as valves that can control both the flow of plasma (and blood) through the filtration apparatus (and kidney) while regulating the glomerular filtration rate. Finally, the granular cells, a specialized subset of smooth muscle cells of the afferent arteriole, secrete renin, the enzyme that controls angiotensin II production. These cells are part of the juxtaglomerular apparatus (JGA).

10
Q

Filtration properties of the glomerulus

A

Here the concentration of a substance in the filtrate formed in Bowman’s capsule relative to its concentration in the plasma, i.e. its filterability, is plotted as a function of the solute’s molecular size. Obviously for a freely filtered substance this value is 1, and for a totally excluded (nonfiltered) substance this number is zero. This molecular sieving process is also called “ultrafiltration”. Note that the dependence of filterability on molecular size is not especially sharp. Thus solutes up to several thousand daltons in size pass freely though the filter and even molecules up to 50,000 daltons have some measurable filterability. However, at around 60,000 daltons, substances do not pass through the filter, thus this value is referred to as the “molecular size cut-off” of the filter. This cut-off value is not accidental, however, since it is just lower than the size of the serum albumins (67,000 daltons). Nonetheless, significant amounts of smaller proteins do get through this filter (about 2 grams per day); these are reabsorbed and catabolized to their constituent amino acids by the tubular epithelial cells.

11
Q

glomerular capillary endothelium.

A

The flow of the filtrate first passes through the glomerular capillary endothelium. Note that this endothelium lacks the usual slit membranes, thus it is known as a “fenestrated” epithelium. On a molecular scale, these holes are large, and they do not present much resistance to the movement of the plasma through them; hence they contribute little to the ultrafiltration properties of the glomerulus, except to exclude circulating red blood cells from entering the other layers.

12
Q

podocytes

A

On the other side of the filter is a sheet of tubular epithelial cells known as podocytes. Instead of the usual cuboidal/columnar structure of typical epithelial cells, these have rounded cell bodies from which numerous “feet” (pedicels) are projected toward the endothelial cell layer. Feet from adjacent podocytes intimately intertwine; further there appear to be slit membranes that connect the feet. Thus it appears that the podocyte layer has the ability to function as a molecular sieve with the properties. Evidence for this view comes from human genetic diseases in which proteins that form the podocyte feet slits are defective and slits do not form; in these patients the glomerulus is very “leaky” and passes large proteins, suggesting that the slits themselves are molecular sieves.

13
Q

the basal lamina

A

However, it also appears that the podocytes help create and support an equally important filtration barrier, the basal lamina. This structure is a thick basement membrane that is secreted by both the endothelial and epithelial cells into the space between them. It is composed of mucoproteins, which are large complexes of acidic sugars attached to protein cores. Similar to the well known nasal mucous (“snot”), on a molecular level this substance appears as a tightly woven mesh-work with holes of the correct dimensions that account for the filtration properties of the glomerulus. Further, the negatively charged basal lamina accounts for the observations that near the molecular size cut-off, highly positively charged macromolecules filter much better than would be predicted, whereas highly negatively charged macromolecules have a much lower than expected filterability. Also, as you will hear in other lectures in this series, a number of pathological conditions selectively affect the integrity of the basal lamina; this invariably results in deleterious changes to GFR. In any case, it seems most likely that both the podocyte feet slit membranes and the basal lamina contribute to the molecular sieving properties in the glomerulus.

14
Q

Derivation of the Starling equation for filtration

A

Here we consider the GFR as a bulk flow of fluid across the glomerulus. Normally this flow extracts about 20% of the plasma as it flows along the glomerular capillaries, hence we say that the “normal filtration faction is 0.2”. How does so much fluid get filtered into the tubule? First, we consider that the GFR is just a special example of a transmural flow, i.e. a flow across a resistive boundary. The magnitude of such flows is simply given by the pressure difference across the boundary divided by the hydrodynamic resistance to flow of the boundary. For our case of GFR: GFR =deltaP/R where ΔP is the pressure across the filtration layers, and R is the resistance to flow. Let’s consider that R does not normally vary, hence we can simply give it a constant value “K”, and GFR = KDP

15
Q

what are the pressures within the glomerulus that drive and resist filtration?

A

As for any capillary bed, filtration out of the capillary is driven by the value of the blood pressure where filtration occurs, i.e. for GFR the hydrostatic pressure within the glomerular capillary, Pgc. This is the only significant driving force for filtration. However, there are two significant forces that oppose glomerular filtration. The first results from the fact that the filtrate must flow in the narrow confines of the tubule. This results in a “backpressure” at Bowman’s capsule, which we denote by Pt. The second force is less obvious, but as you will see is strong indeed; it’s a net osmotic force that by itself would cause fluid to flow in the reverse direction to the GFR, hence the net osmotic force across the filtration apparatus is an opposing, negative force relative to glomerular capillary pressure.

16
Q

How does the osmotic force arise?

A

It isn’t due to the normal tonicity of the plasma associated with dissolved ions and small solutes, since these filter freely across the glomerulus, and thus osmolarity due to freely filterable substances is the same on both sides of the glomerulus. Instead, on the blood side, the large dissolved proteins in the plasma (i.e. serum albumin, immunoglobulins) do not filter, hence as water is extracted from the plasma by filtration, the protein concentration rises, resulting in a net negative osmotic pressure that opposes filtration. We call this pressure the colloid osmotic pressure or COP. Sometimes this pressure is called the oncotic pressure. It is symbolized as πgc.

17
Q

Net Filtration Pressure or NFP

A

we can now write the equation for GFR as: GFR=K(Pgc –Pt-piegc. The sum of the forces (Pgc-Pt- πgc) is called the Net Filtration Pressure or NFP. You will also recognize them as the “Starling forces” that describe glomerular capillary filtration.

18
Q

Actual magnitudes of the forces and the NFP

A

Pgc=46mm, Pt=10mm, PIEgc=30mm. NFP=6mm

19
Q

The potential role of changes in filtration area “A” in GFR regulation

A

The glomerular capillaries are covered by the so-called mesangial cells. It is thought that the mesangial cells can contract, and in doing so can decrease the filtration area, hence GFR. Thus it is possible that GFR to some unknown degree might be regulated or affected by the action of the mesangial cells. However, this principle of GFR regulation has not been fully established. However, an area where changes in filtration area, or indeed specific conductivity ρ are clearly important is in diseases that attack or affect the glomerular apparatus.

20
Q

GFR is regulated to be relatively constant in normal physiology.

A

At this point we should discuss whether changes in GFR can be part of the overall regulatory picture for ECF substances under normal circumstances. For example, suppose a person drank a large excess of water; wouldn’t it make sense to simply increase the GFR while holding water reabsorption constant? This would certainly increase urinary excretion of water. The problem with this approach lies in the fact that the filtration process is nonspecific. Thus when one increases GFR with the aim of putting more water in the tubule, the other regulated solutes are also increasingly filtered; hence to stay in balance for all other regulated substances, one would have to increase the reabsorption (or decrease the secretion) for all of these other ECF components. Naturally this would result in a highly complex and energetically costly way of doing business. Instead, it appears that the renal system tries very hard to keep the GFR constant and then varies the rates of tubular handling of each regulated substance as necessary. For the example above, what happens is that GFR stays relatively unchanged but the rate of water reabsorption is decreased, resulting in the required increased urine output to stay in water balance.

21
Q

The necessity for Pgc regulation

A

Note that under normal circumstances that Pgc is about half of the MAP found in major arteries. Naturally all capillary blood pressures are fairly low compared with MAP since the blood must flow some distance from the major arteries through ever narrowing blood vessels before reaching the capillary beds. Changes to MAP do not cause proportionate changes in glomerular capillary pressure. Instead, Pgc is very tightly regulated by the process of autoregulation.

22
Q

GFR constancy

A

is maintained under normal conditions by glomerular capillary autoregulation. The phenomenon of autoregulation maintains blood flow in capillary beds very constantly in the face of a wide range of changes in arterial blood pressure (MAP). This is done via a myogenic mechanism, i.e. when MAP changes smooth muscle cells of the arteriole very precisely constrict or dilate in order to keep the downstream capillary blood flow constant. The very same mechanism applies to the kidney. Here it is the afferent arteriole that serves as a regulating valve to keep renal blood flow constant. However, the additional aspect of this regulation for the kidney is that with this mechanism Pgc and hence GFR are also maintained constant. This is just a consequence of the simple hydrodynamics of this system. Thus the significant changes in MAP in the renal artery can in theory be exactly compensated by the action of the afferent arteriolar valve, leaving all of the flow and pressures downstream of the valve unchanged.

23
Q

Problems in autoregulation

A

In reality, autoregulation operates over a range of pressure changes and is not entirely “perfect”. Over a fairly wide range of MAP values (generally 75 to 150mm Hg in the human), RBF, Pgc, and GFR all remain fairly constant. However, there is some residual error in the autoregulatory mechanism, resulting in an upward creep of all three regulated variables as MAP increases. Outside the range of regulation, these variables do change rather dramatically. In particular, note how Pgc rises rapidly when MAP exceeds its autoregulatory range. This situation, such as occurs in malignant hypertension, can have grave consequences for the integrity of the delicate glomerular capillaries.

24
Q

Response of the kidneys to severe hypovolemia

A

The autoregulatory mechanisms operate constantly during our normal daily lives, and they are intrinsic to the kidneys. However, we also have a built-in emergency response of the renal system in the rare event that our ECF volume decreases significantly, posing a threat to the function of the circulatory system. One of the vital circulatory responses to hypovolemia is to increase the resistance of the peripheral circulation, thus effectively shunting the remaining cardiac output to the organs essential for short term survival, namely the heart, brain, and lungs. In this scheme the kidneys are called upon to make their own blood flow reduction; this is especially important since the RBF is usually such a high proportion of the cardiac output. On the other hand, it is also important to viability that the kidneys not shut down markedly, but rather continue to process the plasma at a level as close to normal as possible. This is because both the primary pathological event (e.g. K+ release from tissues in muscle trauma or burns) and the recovery phase (e.g. drinking lots of pure water after severe dehydration) can involve threats to ECF homeostasis. However, complete filtration shutdown could easily occur if the volume loss leads to hypotension and a MAP drop below the autoregulatory range. Thus the kidneys have an alternative mechanism that preserves GFR at a reasonable level while also decreasing RBF.

25
Q

The renal solution to hypovolemia

A

GFR is maintained by co-ordinate constriction of the afferent and efferent arterioles. The afferent and efferent arterioles represent a sophisticated valve system that can independently control RBF and GFR. In response to a prolonged and severe hypotension, both the afferent and efferent arterioles constrict, and they do so to about the same degree. First, afferent arteriolar constriction will increase resistance to renal blood flow, hence will have the desired effect of decreasing RBF. However, this will also have the effect of significantly decreasing Pgc and according to the principles discussed above, even more dramatically decreases GFR, perhaps even stopping it. Thus the second constriction of the efferent arteriole has a beneficial effect, for its constriction serves as a flow diverter in which Pgc is restored to its normal value, and GFR resumes a normal flow (to understand this effect better, imagine what would happen if extreme efferent constriction completely shut off blood flow through this arteriole: In this case the only pathway for plasma to flow would be to filter through the glomerulus into the tubule). In addition, the constriction of the efferent arteriole also serves to further increase renal vascular resistance, thus causing RBF to decrease even further, additionally aiding perfusion of the central organs.

26
Q

Affects of renal compensation to hypovolemia

A

In actual practice GFR will decrease somewhat, the degree depending on the severity of the hypovolemia. However, its proportional decrease will always be less than that of the RBF. The reason for the decline of GFR under these conditions relates to the fact that by decreasing RBF, hence RPF, the filtration fraction will rise and thus πgc increases. According to the GFR equation this will tend to reduce GFR even if Pgc is held constant by the coordinate constriction of the flanking arterioles. Note that this doesn’t happen in autoregulation, since RPF is maintained constant, thus filtration fraction stays constant, and πgc is thus unchanged as well. Note also the differences in how the arterioles are affected in autoregulation versus hypovolemia. In particular, for a drop in MAP the autoregulatory response is to dilate the afferent arteriole, whereas the baroreceptor response in hypovolemia causes a constriction of this very same vessel. Naturally, for the hypovolemic response to be effective at some point it has to overwhelm the action of the autoregulatory myogenic response. Such is the case when the hypotension resulting from a loss of volume becomes sufficiently severe and chronic.

27
Q

Regulation of coordinated constriction of arterioles during hypovolemia

A

The coordinate constriction of arterioles during hypovolemia is mediated by external and renal baroreceptors. First, the usual baroreceptors in the main arteries sense a decline in MAP. If this decrease is significant and long-lasting, a baroreceptor reflex occurs in which the activity of the renal sympathetic nerve increases. This nerve forms neuromuscular synapses on the afferent and efferent arterioles, and as with any such activity, causes the arteriolar muscle to contract, thus constricting the arterioles together. This has the desired effects of decreasing RBF while resulting in either an unchanged or more modestly decreased GFR. Second, the external baroreceptor reflex results in a hormonally-mediated constriction of the arterioles, since neural stimulation of the afferent arteriole also causes the increased release of renin from the granular cells of the JG apparatus. This automatically increases the levels of the powerful vasopressor, angiotensin II, which circulates and acts directly to constrict the arterioles of the kidney. Thirdly, the renin/angiotensin axis is stimulated by detection of reduced arteriolar pressure by intrarenal baroreceptors thought to reside on the granular cells themselves.

28
Q

Filtration equilibrium

A

As complex as this may all seem to you, we have still employed the simplifying condition that all filtration takes place at the same point in a glomerulus. In reality, the glomerular capillaries have significant length, and as the plasma travels down this vessel it is constantly losing water to filtration and losing blood pressure due to the resistance of the capillary. Thus, according to equation, the NFP for filtration is constantly decreasing along the capillary length. In fact, under certain circumstances of low RPF, it is possible that NFP will reach 0 at some point before the plasma exits the capillary! This point is known as “filtration equilibrium” and from here on until the end of the glomerular capillary, no further filtration takes place. In effect, in this situation the GFR takes a “double hit”, since not only does filtration start off with a lowered NFP due to hypotension, but the effective area for filtration is reduced since there is now a part of the glomerulus under which there is no filtration. At present it is uncertain whether filtration equilibrium occurs in humans pathologically, but has been shown to occur in certain animals.

29
Q

Role of renal prostaglandins

A

These prostaglandins are produced by the renal interstitial cells that are mainly located in the kidney medulla between the renal pyramids. These prostaglandins are secreted in response to angiotensin II and have a local dilatory effect on the renal arterioles. However, since the role of AgII in the hypovolemic response is to constrict these same arterioles, for what purpose does AgII produce a hormone that antagonizes its own actions? The current thinking is that two important things are accomplished by dilatory renal prostaglandins. The first is that the prostaglandins maintain an adequate renal blood flow by blunting the affect of AgII on renal arteriolar constriction. This is thought to be important since renal tubular cells are highly sensitive to ischemic hypoxia (i.e. lack of oxygen caused by reduced blood flow); thus the prostaglandins provide a measure of protection against acute renal failure in hypovolemia (refer to your pathophysiology lectures for further details). Second, it is believed that the dilatory effect of prostaglandins is somewhat selective for the afferent arteriole. This will tend to restore GFR back toward normal. However, it should be stressed that renal dilatory prostaglandins do not eliminate the hypovolemic mechanisms described above but merely blunt them a bit so that RBF and GFR reductions will not be as severe as they would with the purely vasoconstrictive effects of the baroreceptor and AgII mechanisms.

30
Q

General Anatomy of the kidney

A

A longitudinal section of the kidney shows several regions. It is surrounded by an outer fibrous capsule that has the cortex immediately beneath it. Under the cortex are
the medullary pyramids which are arranged as conical
functional units that consist of a series of tubular elements of nephrons and their associated collecting ducts. Collecting ducts empty urine at the tip of each cone or pyramid (papilla) into a calyx. Calices are essentially urinary drainage conduits that connect various sections of the kidney together at the renal pelvis. Urine leaves the kidney as the ureter in a central region called the hilum. The hilar region is also the site for entry of the renal artery and vein.

31
Q

Vasculature

A

The renal arteries branch from the abdominal aorta, one supplying each kidney. Within the hilar region they branch into anterior and posterior segments that in turn branch into the interlobar arteries that run between the medullary pyramids. Near the medullary-cortical junction they branch into arcuate arteries that run roughly parallel to the outer capsule. They branch further into smaller arteries (interlobular arteries) that course through the cortex toward the capsule. These branch further to form the afferent arterioles that supply blood to the fundamental blood filtration sites, the glomeruli of individual nephrons. The glomeruli are capillary units from which some of the plasma contents are filtered, the remaining blood volume passing into the efferent arterioles that leave the glomeruli. For the glomeruli located in the outer part of the cortex, the efferent arterioles form a capillary plexus that initially surrounds tubules of the nephron within the cortex and can enter the medulla as a capillary plexus called the vasa recta. For the glomeruli that are near the medulla (they form part of the juxtamedullary nephrons), efferent arterioles directly enter the medullary region. Blood from the capillaries drain into radially-oriented interlobular veins, to arcuate veins near the cortical-medullary junction and to interlobar veins that connect to the renal vein leaving the kidney. Blood flow is regulated within the kidney, and the kidney is also involved in regulation of blood pressure.

32
Q

The Nephron

A

The nephron is the basic functioning unit of the kidney, and the two main functions (blood filtration and selective resorption) are carried out by the Renal corpuscles and Renal tubules, respectively. The renal corpuscles are all located in the cortex. A tubule is associated with each corpuscle, and usually courses for at least part of its length into the medullary region where it forms a hairpin bend in a region of the tubule called the loop of Henle. Renal tubules of juxtamedullary nephrons extend into the deepest regions of the medulla, and play a key role in establishing a medullary salt gradient. Tubules of cortical nephrons can extend to different distances in the medulla, but do not extend to the depths of juxtamedullary nephrons.

33
Q

The Renal Corpuscle

A

The renal corpuscle is where filtration of blood occurs. It consists of a condensed capillary network (the glomerulus) surrounded by an epithelial capsule (Bowman’s capsule). Single afferent and efferent arterioles enter and leave the anastomosing glomerular capillaries. In the interior of the capillary bed is the connective tissue of the mesangium containing mesangial cells. On the outside of the capillaries is a special layer of cells called podocytes, which comprise the visceral epithelium of Bowman’s capsule. Between the endothelial cell of the capillary and the podocytes is a crucial basal lamina called the filtration barrier. It has properties unique to the kidney that enable it to selectively filter the blood. The outer layer of Bowman’s capsule (the parietal epithelium) is continous with the visceral epithelium at the base of the glomerulus. However, the outer layer is a simple squamous epithelium and is entirely different in structure from the layer immediately surrounding the glomerulus comprised of podocytes, as will be discussed below. The space in Bowman’s capsule represents the beginning of the urinary space. It is continuous with lumen of the proximal tubule that will ultimately drain urine to a minor calyx.

34
Q

The Filtration Barrier

A

This is the barrier immediately surrounding the blood flowing in the glomerulus, and is comprised of 1) The fenestrated endothelium of the capillary itself, 2) The complex, thick basal lamina and 3) The filtration slits between podocytes. Filtration though this barrier is driven by hydrostatic pressure of the blood. The fenestrated endothelium of the glomerular capillaries themselves prevents formed elements of the blood, including cells and platelets, from coming in contact with the basal lamina. However, all components of the plasma can come in contact with the basal lamina.

35
Q

The basal lamina

A

is thinner where capillaries con- tact the mesangium, which is itself comprised of connective tissue of similar density. The thickness of the lamina is markedly increased in certain dis- eases such as diabetes mellitus, and autoimmune disorders such as lupus. The basal lamina is a barrier to molecules approximately 60-70,000 MW in size, which includes albumin in the plasma, but this barrier is not an absolute number. Negatively-charged molecules of somewhat smaller size will often not penetrate the barrier, although most very small molecules (

36
Q

The function of podocyte filtration slits

A

is unclear, but filtrate must pass through these 25-30 nm slits which have thin diaphragms of about 4-7 nm thickness, and they may be involved in part in the filtration process. The loss of foot processes in certain renal diseases results in excessive plasma protein in the filtrate, but it is not known whether this is a direct result of podocyte absence or reflects an effect of the maintenance of the integrity of the basal lamina.

37
Q

Mesangial cells and the mesangium

A

Within the more central regions of the glomerulus is the connective tissue of the mesangium which supports the glomerular structure. Mesangial cells secrete the matrix that is continuous with the basal lamina in the inner part of the glomerulus where the mesangium meets the endothelial cells of the capillaries. The mesangial cells are phagocytic and are thought to play a role in maintenance of the filtration lamina in the inner portion of the glomeruli. These cells are also contractile, although a potential role for them in contraction of the glomerular cap- illary loops is not clear.

38
Q

The Renal Tubules

A

Most of the urinary filtrate (over 99%) is returned to the blood during the meandering trip from Bowman’s capsule to the renal pelvis. The job of selective resorption of water and many of the solutes of the filtrate occurs as the filtrate passes through the tubules and collecting ducts. Regions of the tubular portion of the nephron carry out specialized resorption functions, and include:
1. The proximal convoluted and straight tubule.
2. The thin descending loop of Henle.
3. The thin ascending loop of Henle.
4. The ascending thick loop of Henle.
5. The distal convoluted tubule
, 6. The collecting tubule. The collecting tubules of several
nephrons join the collecting ducts which also have specialized water-resorption functions. The specialized functions and control of resorptive processes will only be briefly covered here, but will be covered in greater detail in physiology lectures. Details of the cell structures of the tubular endothelial layers en route to the renal papillae are clearly indicative of the types of functions they participate in.

39
Q

The Proximal tubule

A
This portion of the renal tubule is specialized in having a cuboidal epithelium with an extensive brush border of tall microvilli on the luminal side, as is typical of epithelia undergoing major absorptive transport processes. Cells are connected near the luminal
surface by a band of tight junctions. The basolateral side has
extensive basal and lateral infolds
 that are highly enriched in Na+K+
ATPase, for pumping sodium out
the basolateral side. This drives
the uptake of sodium, glucose,
and amino acids by facilitated
diffusion at the microvillar lu
minal side. These cells are
packed with numerous mitochondria for ATP production to sup-
port the extensive uptake of sol
utes in this portion of the tubule.
Roughly 75-85% of the volume
 of the filtrate is absorbed by the
time it enters the thin portion of the loop of Henle.
40
Q

The loop of Henle

A

An abrupt transition occurs between the thick descending portion of the proximal tubule and the thin portion of the loop of Henle. Cells in the thin loops are simple squamous epithelium, with the thin descending and thin ascending portions having different permeabilites to NaCl and water. This permits an osmotic salt gradient to be maintained in the medulla, from the cortical junction to the medullary papilla, which will be discussed in physiology. Cells of the thick ascending loop are cuboidal and active transporters of sodium, and contain numerous mitochondria similar to the cells of the proximal tubule, with extensive basolateral infolds.

41
Q

The Distal tubule

A

The distal tubule is a continuation of the thick ascending loop of Henle after it enters the cortex. This epithelium is similar in appearance throughout.
Cells are cuboidal, with short micromilli
on the luminal side which are not as abundant as in proximal tubule cells. Distal tubules play a major role in acid base balance. They respond to aldosterone and antidiuretic hormone (ADH) in their more terminal regions. Like the cells of the proximal tubule, they have numerous mitochondria, and basolateral infolds. The basolateral regions also contain Na+K+ ATPase to drive the active transport of sodium across the basolateral membrane.

42
Q

Collecting Tubules and Ducts

A

Collecting tubules join the distal tubule to the collecting ducts. Two major cell types are present, the more abundant principal (clear) cells and intercalated cells (which tend to stain more darkly). Cells of the tubule are cuboidal, with very short microvilli, becoming increasingly more columnar as they reach the collecting duct, where the cells are columnar. Principal cells are active transporters, and couple sodium uptake to potassium secretion into the lumen of the tubules. Intercalated cells function to secrete H+ and reabsorb bicarbonate; hence their main function is to regulate acid-base balance. Water resorption is regulated in the collecting tubules and ducts through the action of antidiuretic hormone, which increases the permeability to water of the collecting ducts and tubules through an increase in aquaphores (specific proteins permitting water passage) in the membrane. This results in a more concentrated urine. Collecting ducts join with other collecting ducts that run vertically through the medulla to form large papillary ducts that drain urine into the minor calices.

43
Q

The Juxtaglomerular Complex or Apparatus

A

The juxtaglomerular complex is a specialization of cells located at the vascular pole of the renal corpuscle. The distal convoluted tubule is physically connected to the region near the vascular pole through the macula densa. This is comprised of a specialized set of cells of the distal convoluted tubule that are taller and have larger nuclei. Juxtaglomerular cells are modified smooth muscle cells in the wall of the afferent arteriole. These cells can secrete renin. Lacis cells are found in direct contact with the macula densa and juxtaglomerular cells. They have numerous processes and continuity with the mesangium. Juxtaglomerular cells contain granules of renin, an enzyme that cleaves a plasma protein angiotensinogen to angiotensin I. Angiotensin I is primarily converted to Angiotensin II by the lung. It is a potent vasoconstrictor. So juxtaglomerular (JG) cells control both systemic and local blood pressure through renin release. This topic of blood pressure control will be discussed in greater detail in physiology lectures.

44
Q

the cells of the macula densa

A

Another control mechanism is that the cells of the macula densa are thought to respond to the concentration of chloride ions (referred to quite often in texts as responding to sodium ions or osmolarity) in the urinary filtrate pre- sent in the distal tubules. Some of the cells of the macula densa are in close contact with JG cells. Lacis cells are located between the macula densa and cells of the mesangium. Although no clear role has been demonstrated for them, they have been suggested to participate in some way in a feedback signaling event between cells of the macula densa and intraglomerular mesangial cells.

45
Q

Interstitial cells

A

Other cells are present outside of the tubules in the medulla and to a lesser extent, in the cortex. Wandering macrophages are present, as well as fibroblast-like cells. Stellate cells containing lipid droplets are pre- sent. Some interstitial cells and some tubule cells have been found to contain mRNA for erythropoietin, which, as you may recall, is produced in the kidney in response to hypoxia.

46
Q

Gross anatomy of the kidney

A

Blood vessels and the ureter enter through a zone called the hilus. A renal pyramid contains a group of nephrons that drain into a minor calyx as urine proceeds to the ureter. Minor calices connect to form major calices of the renal pelvis. The cortex and medulla are indicated.

47
Q

components of the nephron

A

Blood flows via the afferent arteriole into the glomerulus of the kidney. Urinary filtrate enters the urinary space of the renal corpuscle, and then into the proximal convoluted tubule. Components of the filtrate are reabsorbed with specialization along the route to the ureter, first in the proximal tubule, the straight descending segment of proximal tubule, the descending and ascending thin segments (the loop of Henle) the straight segment of the distal tubule, and the distal convoluted tubule. Note that more than one distal tubule joins with the collecting tubule, which drains concentrated urine into the minor calyx of the kidney. In some juxtamedullary nephrons a short arched duct is interposed between the distal tubule and the straight collecting duct. The tubular portion of the nephron begins as the parietal layer of Bowman’s capsule and terminates at the end of the distal tubule.

48
Q

Cortical and juxtamedullary nephrons

A

In each human kidney there are an estimated one million or more nephrons. About 1/7th of these are juxtamedullary nephrons; the renal corpuscles of these nephrons lie deep in the cortex, adjacent to the medulla. Juxtamedullary nephrons have long loops of Henle. Cortical nephrons have short loops of Henle or none at all. The functional significance of the loop of Henle is that the length of the medullary component is related to their ability to generate more concentrated urine

49
Q

The two primary constituents of the ECF

A

water and sodium chloride. The maximal excretion of either salt and water is only a small fraction of the filtered load. In other words, under normal circumstances most of the filtered load of water and salt is obligatorily reabsorbed, with only a small fraction under homeostatic control. In percentage terms, at least 86% of the filtered water and 98% of the filtered NaCl must be reabsorbed. Thus the regulatory mechanisms that we will describe affect only 14% of the filtered water and 2% of the filtered NaCl.

50
Q

Physiological implications of water and NaCl filtration, excretion, and reabsorption

A

nearly all of the obligatory recapture of water and salt occurs in the “proximal segments” (i.e. proximal tubule and loop of Henle), while the homeostatically varied reabsorption primarily takes place in the so-called “fine tuning” segments, namely the distal tubule and collecting duct. This spatial separation and arrangement of obligatory reabsorption occurring first with regulated reabsorption later is necessary for the efficient handling of water and salt.

51
Q

Pathological implications of water and NaCl filtration, excretion, and reabsorption

A

We might briefly consider whether these ranges, in particular the maximum excretion rates, are adequate to defend the ECF against pathological gains in water or salt due to over-ingestion. Water ingestion in normal folks generally presents no problem, for it is behaviorally (and anatomically) very difficult to ingest a large volume of water sufficient to overwhelm the excretory abilities of the kidneys in maintaining water balance. However, in certain pathological conditions, for example impaired GFR or inappropriately elevated water reabsorption as might occur with use of the drug “ecstasy”, retention of excess ingested water can occur, leading to the dangerous condition of water intoxication. On the other hand, for salt intake in the normal individual, the regulated component is so small, that it is indeed possible for dietary intake to exceed excretory limits. Thus the average daily salt intake in the U.S. is about 9-10 grams, and given the prevalence of high salt foods in our diet it is likely that some individuals regularly exceed the upper excretory limit. This has suggested to some researchers a partial explanation for the epidemiologic link between high salt and essential hypertension. As we will discuss later, the addition of salt to the ECF has a volume-expanding effect, which in itself would increase blood pressure. However, the actual pathology must be more complex, for the hypertension tends to persist even when the salt-induced volume overload is corrected. No doubt you will hear more about this subject in your clinical lectures on hypertension.

52
Q

Energy-requiring transport of sodium

A

Na+ transport is the process that drives nearly all other transport phenomena in the tubule. The primary energetic event occurs when Na+ is actively extruded from the interior of the tubular epithelium by the basolateral Na+/K+-ATPase ion pump. As you will recall, in all living cells the effect of this pump is to greatly reduce the sodium concentration inside the cell by moving it to the extracellular fluid. In the renal tubular epithelial cell, since these pumps are located on the basolateral membrane, sodium is moved to the serosal side of the epithelium. Also remember that this pump also enriches the cell interior in potassium, which selectively leaks out of the cell to establish the cell interior at a negative electrical potential. In the tubular epithelial cells, this creates a large driving force for Na+ entry into the cell; this happens when sodium passes from the lumen into the cell through sodium channels in the apical membrane. Thus in the overall reabsorption scheme for Na+ there is first passive lumenal movement of the ion down its electrochemical gradient into the cell followed by basolateral pumping of the ion into the serosa.

53
Q

Reabsorption of chloride, water, and other solutes

A

are coupled to the active reabsorption of sodium. Since sodium is a cation, its primary movement requires the associated co-transport of an anion or the counter-transport of a cation to maintain electrical neutrality. Mostly this is accomplished by the reabsorption of chloride. The most common mechanism for this chloride movement is paracellular, i.e. through the tight junctions. Now both sodium and chloride accumulating on the serosal side of the membrane. This creates an osmotic gradient across the epithelium. Thus, if a pathway for water is provided, water will then also move from the lumen into the serosal interstitium. In the renal epithelium, such movement may be either paracellular or transcellular or both, depending on the tubular segment. Finally, the primary electrochemical gradient for lumenal sodium entry can be utilized by other solutes in a coupled-transport scheme. Here we note that glucose can be reabsorbed in a “secondary” active transport scheme in which the movement of glucose from lumen into the cell is driven by coupling it to the energetically downhill movement of Na+ via the Na+ /glucose co-transporter. The utilization of energy from the Na+ gradient in this process actually allows glucose to be concentrated in the cell relative to its serosal and lumenal values, hence it can then passively move from the cell to the serosa via a passive carrier mechanism. Such a reabsorptive process is essential for the recapture of nonspecifically filtered glucose as well as filtered amino acids. In summary, the primary, energy-requiring basolateral movement of Na+ allows the coupled movements of most other reabsorbed ECF components.

54
Q

The role of the proximal tubule

A

Taking an obligatory “big bite” of the filtered load of water and NaCl, and the recapture of important metabolites in the filtrate. In the proximal tubule 65% of the filtered water and NaCl is obligatorily reabsorbed. Thus this segment always reabsorbs the majority of the filtered load for these two substances, regardless of homeostatic requirements. The water “pores” in the transcellular pathway are actually proteins known as “aquaporins”. The proximal tubule also plays an essential role in the recapture of metabolites that have been incidentally filtered into the tubule because of their small size. Thus essentially all of the filtered glucose and amino acids are reabsorbed here, using Na+ - driven co-transporters described above. Most of the filtered bicarbonate, which is essential for the buffering of metabolic acid, is also reabsorbed here, again involving Na+ as a cofactor. Thus much of the obligatory Na+ reabsorption in the proximal tubule occurs in conjunction with small organic molecule recapture. Thus it seems that the kidney is no procrastinator, but has evolved to recapture these vital metabolites at the first opportunity. This is an important point, since due to the high rate of filtration the failure to recapture even a small fraction of the filtered amount of any metabolite would result in the excretion of a large mass of it over the course of a day.

55
Q

Transport maximum

A

since recapture is mediated by discrete transporters, the capacity for reabsorption of these metabolites in the proximal tubule is finite. This maximal absorption rate is often referred to as the transport maximum for a substance. For glucose, the transport maximum for reabsorption can be exceeded if the plasma concentration of glucose becomes abnormally high, as in diabetes mellitus. In this case, the rate of filtration can indeed exceed the transport maximum, and thus some glucose escapes reabsorption to appear abnormally in the urine. Thus “glucosuria” is one of the diagnostic indicators of diabetes mellitus. In any case, since equivalent proportions of NaCl and water are reabsorbed in this segment, the tubular fluid remains approximately isotonic during its journey through the proximal tubule. This is because the reabsorptive pathway for water is very permeable and close to the pathways of sodium chloride and metabolite reabsorption, thus water flows freely and essentially achieves osmotic equilibrium across the epithelium of this segment. This is not the case for the loop of Henle, the next segment encountered buy the filtrate.

56
Q

The loop of Henle

A

creation of a hypertonic interstitium. As noted above, a greatly reduced but still isotonic filtrate enters the loop of Henle. Here, however, the processes for NaCl reabsorption are quite different than in the proximal tubule and they are spatially separated from water reabsorption in this segment. The primary event of NaCl reabsorption occurs in the ascending limb of the loop; this reabsorption is robust in that it transports 25% of the original filtered load from the lumen into the surrounding interstitium. Naturally this movement of solute into this space makes the interstitium highly hypertonic, creating a significant osmotic gradient for water reabsorption.

57
Q

the water permeability of the ascending limb

A

is quite low, so water does not follow salt from the lumen in this segment. Instead, note that the descending limb of the loop has high water permeability and no permeability to NaCl; thus water flows fairly readily from this segment into the interstitium in response to the osmotic gradient set up in the ascending limb. However, the relative water permeability and tubular flow rate of the descending limb is such that osmotic equilibrium does not occur in this segment, but rather only 15% of the original filtered load of water is reabsorbed. Since overall more NaCl than water is reabsorbed, a hypertonic interstitium and a hypotonic tubular fluid are created. Further, as in the proximal tubule, these processes are largely obligatory. The hypertonic interstitium is a crucial component of regulated water reabsorption in the fine tuning segments.

58
Q

The cellular mechanisms for NaCl reabsorption in the ascending limb

A

are a bit different than those of the proximal tubule. The main difference lies in the apical membrane transport of NaCl, which involves a Na/K/2Cl co- transporter (actually these are in the thick part of the ascending limb, but we’ll not bother with this distinction further). As with glucose transport, this co-transporter secondarily couples the energy in the sodium gradient to drive the uphill reabsorption of potassium and chloride. As such it is a significant mechanism also for the reabsorption of filtered potassium, a fact that will assume more importance when you learn about the potassium wasting side effects of so-called loop diuretics.

59
Q

Reabsorption of water and NaCl in the distal tubule and collecting duct (“fine tuning” segments)

A

the combined obligatory reabsorption in the proximal tubule and loop accounts for nearly all of the required unregulated reabsorption. Thus only 20% of filtered water and 10% of filtered NaCl remain to enter the distal tubule at this point. While there is still some obligatory reabsorption that occurs in these segments (i.e. 6% of filtered water, 8% of filtered NaCl), we’ll concentrate on the regulatory mechanisms. On the surface, these seem very similar to the basic mechanism of tubular epithelium that applies to the proximal tubule. However, there are several critical differences that allow the reabsorption of salt and water to be homeostatically varied here. First, the tight junctions in these segments are indeed “tight”, i.e. the majority of reabsorption must use the transcellular pathway that involves selective transporters. Second, although there is a high basal rate of reabsorption, the rate of transport can be varied by circulating hormones.

60
Q

NaCl handling

A

The basic pathway for sodium reabsorption here is upregulated by the mineralocorticoid hormone aldosterone. Aldosterone effect is to increase the number of apical sodium channels and basolateral sodium pumps, thus the cellular rate of sodium transport increases. In addition, these is also evidence that aldosterone increases ATP synthesis, the primary fuel for the process. How does aldosterone do this? The fundamental mechanism occurs when aldosterone enters the cell (it is a lipid soluble hormone and easily crosses the cell membrane from the blood) and binds to an intracellular receptor. The hormone/receptor complex then diffuses to the nucleus, where it turns on genes that increase the synthesis of transporter proteins. Naturally this is not an instantaneous process, but rather takes place on the scale of an hour or more. However, there is some indication of a faster increase in reabsorption caused by aldosterone that occurs in tens of minutes. This is thought to involve the rapid insertion of pre-existing pools of transporters that reside in small vesicles near the cell membrane.

61
Q

Water reabsorption

A

Overall, both the obligatory basal reabsorption of salt (8% of the original filtered load) and the aldosterone-controlled variable transport (up to 2% of the original filtered load) add more solute to the interstitium surrounding the fine tuning segments. Since this interstitium was already made hypertonic by the action of the ascending loop, it becomes even more so, thus increasing the osmotic gradient between the lumen of the fine tuning segments and the interstitium. Thus there is a high driving force for water reabsorption. In the basal state these segments are fairly water impermeant, so relatively little water will flow across them. However, the water permeability of these segments can be dramatically increased by the actions of the peptide antidiuretic hormone (ADH) (also known as “vasopressin”). In the presence of ADH, small vesicles containing aquaporins are fused into the apical membrane of the epithelial cells. Since there are pre-existing, unregulated aquaporins in the basolateral membranes, this completes a potent transcellular pathway for the osmotic flow of water from the lumen to the serosa. The intracellular signaling pathways for ADH action have been well worked out and involve the initiation of an intracellular phosphorylation cascade when ADH binds to a receptor on the basolateral membrane. This pathway also initiates the de novo synthesis of aquaporins. The response to ADH is very rapid indeed, and similarly the lifetime of the aquaporins in the apical membrane is very short due to rapid turnover. Thus unlike the response to aldosterone, the regulation of water reabsorption is a very acute and fast responding system.

62
Q

A basic integrated description of the nephron handling of water and NaCl

A

While in the proximal tubule most of the filtered load is obligatorily reabsorbed in an isotonic fashion, in the loop of Henle the spatial separation of NaCl and water reabsorption results in the creation of a hypertonic interstitium and a large osmotic gradient across the tubular epithelium of the fine tuning segments. In the fine tuning segments further NaCl reabsorption occurs both obligatorily and homeostatically in response to the hormone aldosterone. Either of these processes will further increase the trans-tubular osmotic gradient. Finally, ADH controls the water permeability of the fine tuning segments and in this way regulates the reabsorption of water using the interstitial osmotic gradient.

63
Q

How reabsorbed water and NaCl flows from the renal interstitium to the peritubular capillaries

A

Starling forces for bulk recapture flow. There is one additional aspect of basic nephron function to be considered, namely how does the huge amount of water and NaCl pumped across the tubular epithelium actually re-enter the capillaries to be returned to the ECF? In actuality, this process involves a bulk flow of interstitial fluid into the peritubular capillaries, and as such it is governed by the same Starling principles for the other capillaries flows that you have learned about (e.g. the GFR). As discussed for GFR previously, this is another transmural flow, and the Starling equation for it (which we’ll simply designate as Fic) is as follows: Fic = K’ (Pint + πcap– Pcap - πint) where K’ is again a constant representing the hydraulic conductivity of the pathway over which flow occurs. What we are interested in here is knowing what forces actually drive this flow.

64
Q

The flow dependence of tubular processing and it implications

A

Protecting the fine tuning segments. Up to this point we have described a highly simplified scheme in which filtration of water and its dissolved NaCl is held constant, followed by constant reabsorption of most of the filtered load in the early tubule, and then is“fine-tuned” by a variable reabsorption/excretion of these substances to achieve overall ECF balance. However, there are circumstances, particularly in pathological situations, where this scheme is upset. Many of these have to do with changes in tubular flow, so we need to explain why this can have a profound effect on the tubular processing and renal excretion of salt and water.

65
Q

The reabsorption and excretion of NaCl and water can be affected by tubular flow changes

A

To understand how flow rate affects the urinary excretion rate of a reabsorbed substance, we should view the tubular journey of the filtrate as one in which filtered salt and water stream past a gauntlet of reabsorptive mechanisms in an attempt to move to the outside world in the urine. Thus the faster fluid flows in the tubule the less time it has to interact with the epithelial transporters that reabsorb it; thus increased tubular flow tends to allow a greater proportion of tubular substances to escape reabsorption, thus their excretion rate will tend to increase with tubular flow. Naturally the opposite is true for reduced tubular flow rates, i.e. exposure time of tubular substances to their transporters is increased, hence a greater proportion of them is reabsorbed, thus their urinary excretion rate decreases. Tubular flow also dramatically affects the excretion of secreted substances such as potassium, although the reasons for this are somewhat different.

66
Q

Diuretics

A

increase tubular flow and excretion of most substances. Diuretic substances are those that increase urine output, and all do so in large part by decreasing water reabsorption in some part of the tubule. This results in a larger volume of water staying in the tubule, hence an increased tubular flow results. A consequence of this is that all solutes in the tubular fluid from this point on flow faster past their transporters, hence their excretion will tend to increase also. Thus most diuretics tend to increase the urinary excretion rate of sodium, potassium, and chloride indirectly due to this flow effect.

67
Q

Consequences of filtration changes for the regulation of NaCl and water

A

In the above discussion, we have assumed that filtration is being held constant by autoregulatory mechanisms described in lecture #2. Now we consider that imperfections in the autoregulatory control of GFR as well as a number of pathological conditions can indeed result in changes to the filtration of the plasma outside of the normal range. Naturally such changes will change tubular flow from at the very start of the tubular stream. As such, unregulated variations in GFR will then tend to cause unregulated changes in the excretion of salt and water. But how serious can such an effect be? What might happen if, for example, imperfect autoregulation resulted in a very small, 1mm increase in Pgc at the glomerulus? This would raise the NFP from 6 to 7mm, hence GFR would be expected to rise by about 17%. While significant, one might assume that since this change is a small fraction of the original GFR that it wouldn’t pose much of a problem for the tubular system to compensate for. The same arguments can be made for NaCl, hence the most modest of regulatory increases in glomerular capillary pressure will tend to completely swamp tubular regulatory reabsorption and cause disastrous increases in urinary excretion.
Finally, one can also consider the effects of decreased GFR in the same way. Here, if obligatory reabsorption remains constant a 1mm decrease in Pgc would result in complete obligatory reabsorption of the filtered load of NaCl and water, hence no excretion of these substances could occur to achieve ECF balance!

68
Q

Defending the “fine tuning segments” against changes in tubular load due to GFR changes

A

In fact, in the circumstances above, there indeed are other compensatory mechanisms within the tubule that greatly blunt the effects of GFR changes to maintain the delivery of water and NaCl to the fine tuning segments very constant. These mechanisms are known respectively as glomerulotubular balance and tubuloglomerular feedback (these very different processes are unfortunately similarly named).

69
Q

Glomerulotubular balance

A

refers to the ability of the obligatory reabsorption mechanisms in the proximal tubule to compensate for changes in filtered load. This compensation is conceptually simple, i.e. proximal tubule reabsorption readjusts to filtration changes so that a fixed proportion of the filtered load of water and NaCl is always reabsorbed. This proportion is 65%. Thus in the example of increased filtration above, nearly two thirds (i.e. 22 liters) of the additional 32 liters of filtrate formed daily would be reabsorbed in the proximal tubule. Naturally this also applies to reduced filtration rates.

70
Q

Tubuloglomerular Feedback (TGF)

A

However, while this glomerulotubular balance mechanism greatly reduces the difference in the filtrate volume from normal, there will still be a surplus or deficit in the tubular filtrate as it flows out of the proximal tubule. This remaining difference from normal filtrate is then dealt with by the process of Tubuloglomerular Feedback (TGF). This mechanism actually directly regulates the GFR of each nephron in response to changes in NaCl concentration at a specialized group of epithelial cells called the macula densa. These cells are in direct contact with cells of the afferent arteriole and they can cause the arteriole to constrict or dilate. In addition, the macula densa cells are placed at the start of the distal tubule. Thus they are perfectly placed to monitor the status of obligatory reabsorption just before the tubular fluid enters the fine tuning segments.

71
Q

the way TGF works

A

A rise in GFR causes an initial rise in tubular fluid flow in the initial segments. Although part of this will be compensated for by glomerular tubular balance in the proximal tubule, the uncompensated part causes fluid to move faster in the loop of Henle. As discussed above, the resultant flow effect causes a reduction in the proportion of NaCl reabsorbed in the ascending limb, hence NaCl concentration increases in the lumen of the ascending limb. As this fluid exits the loop, it immediately encounters the macula densa; the macula densa cells have a special mechanism that senses the rise in NaCl concentration, and through a series of intracellular and paracrine steps they signal the afferent arteriole to contract. This drops Pgc just enough to return GFR to its normal level. Naturally this feedback loop works in the opposite way for an initial decrease in GFR.

72
Q

GFR changes can be significant in abnormal physiology

A

The above discussion demonstrates that in normal circumstances GFR is very tightly regulated and most of the regulation of NaCl and water takes place in the fine tuning segments under hormonal control. However, in a number of pathological circumstances GFR will be altered and be an important part of the pathophysiology.

73
Q

Regulation of ECF sodium levels via aldosterone

A

Naturally, as a cation Na+ must exist with an anion, which is mostly chloride (Cl-). However, a significant amount (~25mM) of this balancing anion is bicarbonate (HCO3-), with lesser amounts of nonvolatile acid and protein anions. In any case, when we talk about aldosterone-mediated Na+ reabsorption, we are really describing a process that mostly co-transports Cl- .

74
Q

Nature of the ECF sensors for Na+ levels

A

The concentration of sodium changes relatively little when sodium is lost or gained from the ECF. This is because sodium is the major osmotic substance in the ECF, and because the ECF is in osmotic equilibrium with a much larger compartment, the cells. Thus losses or gains in ECF sodium actually cause greater changes in ECF volume than they do in sodium concentration. For example, an increase in total ECF sodium mass of 10% is caused by the ingestion of salty food. It might be thought that this would raise sodium concentration in the ECF, but it also raises ECF osmolarity. Since the membranes of most cells are freely permeable to water, water flows rapidly from the cells to the ECF in response to this sodium gain to quickly balance the osmolarity between the two compartments. In a closed system such as this (i.e. fixed volumes of the cellular and ECF compartments), such water flow not only decreases the osmolarity of the ECF but also increases the osmolarity of the cellular compartment as water leaves it; thus equilibrium is reached when the osmolarity of these two compartments converge and become equal. The trick here is to understand that since the cellular compartment is nearly twice as large as the ECF, the flow of water from the cells changes the osmolarity of the ECF more rapidly than that of the cells. To understand this better, it’s useful to think theoretically of the extreme situation in which the cellular compartment is infinitely large; here the flow of water from the cells won’t change cellular osmolarity at all, so all of the change must take place in the ECF. In our example with a 10% addition of salt, this means that ECF volume must increase by 10%, and in the end there in no change in sodium concentration, just a pure increase in ECF volume. In our “real” example, the cell volume is not infinitely large, but “only” twice that of the ECF volume. Thus water flow into the ECF changes its osmolarity twice as fast as water leaving the cells does, hence the osmolarities become equal when the ECF volume has increased proportionally twice as much as the cell volume has. For the specific example of starting addition of 10% of the total salt mass to the ECF, final osmotic equilibrium is reached when the ECF volume has increased 6.7% and the cell volume decreased 3.3%. In addition, note that at this equilibrium the final ECF sodium concentration will only have risen 3.3%, or only half the change in the volume!

75
Q

Effect of two fold greater volume of cellular over the ECF compartment

A

Due to the two-fold greater volume of the cellular over the ECF compartment, gains (or losses) of sodium from the ECF result in two-fold greater changes in ECF volume than they do in ECF sodium concentration. Thus it make sense that the sensors of sodium regulation monitors changes to ECF volume, not sodium concentration. Naturally the convenient volume sensors that we have in the ECF are those for blood pressure, and for sodium they are those that monitor the mean arterial pressure (MAP of the major arteries. The reasoning for the same principle as applied for sodium losses from the ECF is simply the reverse of that given for our example of sodium gains.

76
Q

The feedback loop for sodium regulation

A

For example, with ECF sodium loss, the resultant decrease in ECF osmolarity causes a water shift into the cells, hence ECF volume and MAP decrease. We now want increase sodium reabsorption to increase to prevent undue urinary loss of sodium, i.e. to conserve the remaining sodium in the ECF. Working backwards, we know that we thus need to increase aldosterone secretion, so we need to describe how one gets from a decrease in MAP to an increase in aldosterone plasma levels. First, as we already know, a decrease in MAP activates the renin/angiotensin axis, i.e. external and intrarenal baroreceptor mechanisms cause increased renin release from the JGA/granular cells which then results in elevated levels of angiotensin II. In a function apart from its vasoconstrictive role, AgII acts on aldosterone-secreting cells of the adrenal cortex and causes them to increase aldosterone synthesis (since aldosterone is very membrane permeant, increased synthesis automatically causes increased aldosterone secretion into the blood). Finally, aldosterone then circulates to the epithelial cells of the fine tuning segments and up-regulates the reabsorbed of sodium.

77
Q

ECF sensors for water

A

As for sodium, we start by describing how water levels are physiologically detected. Actually, here we’ll simply state the obvious, that since water is the most abundant component of the ECF, its relative abundance determines ECF volume directly, and it determines ECF osmolarity in partnership with sodium and its anions. Thus it is no surprise that the regulation of ECF water levels involves monitoring of both ECF volume and osmolarity.

78
Q

Feedback loops for volume and osmolarity-mediated control of water reabsorption

A

Since we already know that ADH controls water reabsorption, our object here is to describe the pathways that control the secretion of ADH into the blood. First, ADH is a peptide hormone that is synthesized in neurons located in the hypothalamus of the brain. However, the synthesized hormone is packaged into vesicles and transported down the axons of these neurons which terminate in the posterior aspect of the pituitary gland. It is from the posterior pituitary that the peptide is secreted into the circulation. As one might expect, the reduction in ECF volume in severe sweating is monitored by changes in blood pressure, which to a lesser extent is done by the usual arterial high pressure baroreceptors. However, filling pressure in the left atrium of the heart is considered more important, since atrial distension is a rather sensitive indication of circulating volume (i.e. preload). Thus baroreceptors mainly residing in the auricle (a.k.a. “left atrial appendage”) sense a decreased filling of the left atrium. Then, through a baroreceptor reflex involving neural circuitry that you will learn about next year, the ADH-synthesizing hypothalamic neurons are activated to release ADH from their terminals in the posterior pituitary. The hormone then flows through the circulation to the kidney, where as we’ve described it induces the insertion of aquaporins into the apical membrane of epithelial cells of the late distal tubule and collecting duct, causing enhanced water reabsorption. How about the effect of severe sweating on ECF osmolarity? Here it might be a surprise to learn that normal sweat in actually about 80mM NaCl, and is thus hypotonic. Thus severe sweating remove hypotonic fluid from the ECF, which will become hypertonic. The increased osmolarity of the ECF directly activates brain osmoreceptors on neurons in the hypothalamus; these neurons in turn activate the same ADH-synthesizing neurons in the volume-regulated pathway, with the same result of increased ADH secretion and increased water reabsorption.

79
Q

Volume versus Osmolarity

A

Insights into the true nature of ECF water control. The example of water regulation is relatively straightforward and was chosen to illustrate the cooperative interaction of both the volume and osmolarity- monitoring feedback loops. However, there are circumstances where these separate loops are in opposition to one another. Diarrhea represents an isotonic loss of ECF via pathological secretion into the gut. Here the sufferer loses 3 liters during the acute stage of the disease. 3 liters represents a very severe volume loss for an average individual, and one that would cause the enhanced secretion of ADH according to the left atrial filling pressure pathway in Figure 3. On the other hand, since the fluid loss is isotonic, the osmotic pathway is not activated or inhibited at this stage. In this example we consider first the dehydration of a patient suffering from severe diarrhea. As you will learn in your G.I. lectures diarrhea represents an isotonic loss of ECF via pathological secretion into the gut. Here the sufferer loses 3 liters during the acute stage of the disease. 3 liters represents a very severe volume loss for an average individual, and one that would cause the enhanced secretion of ADH according to the left atrial filling pressure pathway. On the other hand, since the fluid loss is isotonic, the osmotic pathway is not activated or inhibited at this stage. As the patient recovers, he drinks 2 liters of water as he feels a bit better. Since 3 liters of ECF volume were originally lost and only 2 liters have been ingested, the ECF volume will still be decreased by 1 liter. Hence one might expect that there will still be a volume stimulus for ADH secretion. However, our patient has replaced 3 liters of isotonic fluid with 2 liters of pure water, hence the overall osmolarity of the ECF will fall significantly. This should cause an inhibition of the osmotic pathway for ADH secretion, hence ADH levels should fall.

80
Q

how the secretion of ADH varies with changes in either volume or osmolarity.

A

it is apparent that if ECF volume is held constant at its normal value (e.g. 15 liters for our normal human), that changes in ECF osmolarity cause monotonic, roughly linear changes in ADH levels in the plasma for osmolarity values both above and below normal. On the other hand, for normal osmolarity, it seems that changes in ECF volume have little effect on ADH levels, except when ECF volume falls severely. For the initial acute condition of loss of 3 liters of isotonic diarrhea, it is clear that the proportional ECF volume reduction (i.e. 20% loss of ECF volume) is severe enough to trigger massive ADH secretion (see red arrow in figure 5), and since the osmolarity pathway is not activated, a net rise in ADH levels occurs at this point. However, after drinking 2 liters of water during recovery, the volume change is now much more modest (about a 7% decrease, green arrow), and the volume curve predicts that as far as volume goes there will be no stimulus to ADH secretion. But what about the osmolarity input? An ECF in which 2 liters of water were added to 12 liters of isotonic fluid would be reduced in osmolarity by about 15% below normal, hence would be strongly inhibitory to the osmotic pathway. Overall, then, in this example the osmolarity effect “wins” and ADH would be suppressed to very low levels (i.e. near zero), and urinary water excretion would be at a maximum.

81
Q

ECF water regulation

A

is primarily an osmoregulatory system with an emergency low-volume override. Thus, in relatively normal, day-to-day physiology, water is regulated almost entirely to achieve osmotic constancy with little regard for even fairly moderate volume losses. However, when volume loss starts to become severe (.e.g >10% volume reduction), then the volume input rapidly dominates osmotic effects on ADH levels. This low volume dominance of ADH secretion ensures that the circulation is defended at all costs during severe hypovolemia. This important principle is fundamental to understanding disease mechanisms in a number of situations where the circulation is compromised. As opposed to the volume effects, it should be mentioned that the osmoregulatory loop is exceptionally sensitive to very small changes in osmolarity from normal. Thus changes in ADH levels can be detected for osmolarity changes of

82
Q

Regulation of volume overload by atrial natriuretic peptide (ANP)

A

There is no suppression of ADH if volume increases above normal. However, inappropriate ECF volume expansion is potentially a very unhealthy situation, causing hypertension and an increased risk of stroke. For many years after the role of ADH in water conservation was understood, researchers wondered whether a natural diuretic hormone existed that could deal with the opposite limb of water regulation, namely too much ECF water. We now know that such a diuretic hormone does exist and it plays an essential role in the big picture of water regulation. It was initially discovered in prominent granules of atrial cardiocytes; its diuretic role was suggested when these granules disappeared in volume-expanded animals. The hormone was then purified from isolated cardiocyte granules and shown be a potent diuretic peptide that also increased sodium excretion. Hence it has been called atrial natriuretic peptide (ANP; a.k.a. atrial natrituretic factor (ANF); atriopeptin).

83
Q

The pathway and site of action of ANP

A

First, increased ECF volume results in an increased distention of the atria, and this causes release of the ANP granule contents from the atrial cardiocytes. In fact, the granules contain the 52 amino acid pro-ANP form, which is then cleaved by peptidases in the blood to the active 28 amino acid ANP. Active ANP then reaches it targets throughout the body, all of which aid in the increased production of urine. Note how widespread and comprehensive the action of ANP is in antagonizing the normal tubular mechanisms of water reabsorption. Thus ANP acts to increase filtration by selectively dilating both the afferent and efferent arteriole (why will this increase GFR?), placing more fluid in the tubule and increasing excretion of water and sodium through flow effects. At the same time, ANP acts to decrease the sodium and water reabsorption mechanisms that we discussed in lecture #3. First, through a direct action on the brain, it decreases the secretion of ADH. Second, it inhibits renin secretion, hence AgII formation, thus ultimately decreasing aldosterone synthesis. So both hormones that stimulate water and salt reabsorption are inhibited. Further, ANP also prevents the effects of remaining levels of ADH and aldosterone on the tubular epithelium. Both of these actions on hormones will further increase the diuretic effect since they increase tubular fluid flow while they decrease the osmotic gradient for water reabsorption and the water permeability in the fine tuning segments. The result is a massive diuresis with an accompanying increase in sodium excretion, i.e. natriuresis.

84
Q

pathways for water and sodium

A

We have greatly simplified the descriptions of how water and sodium are regulated, for we have treated each substance as if it were regulated independently. This is clearly NOT what happens in real life physiology, as it should be obvious that many of the stimuli and pathways for water and sodium are interdependent. For example, in the severe sweating example discussed above, the individual has also lost a significant amount of sodium, and ultimately the sodium homeostasis pathways must be activated to allow increased the increased reabsorption of sodium over time to replenish what has been lost. This is accomplished in part by the hypovolemically-induced decrease in MAP that turns on aldosterone-mediated Na+ reabsorption. This in turn will allow the retention of dietary sodium in the ECF, which will keep osmolarity high, which then keeps ADH levels up; thus salt and water are restored in proportion to the ECF until the osmotic and volume stimuli attenuate due to return to normal of ECF parameters. In addition, as you will learn in the neuroscience block, AgII acts on the brain to increase thirst, hence motivating the patient to add fluid to the ECF via ingestion.

85
Q

Chronic renal disease and mental illness

A

The burden of physical illness is an important risk factor for suicide independent of mental illness. Increased risk of suicide in patients with chronic renal disease is similar in magnitude to suicide risk with other chronic illnesses such as chronic pulmonary disease and stroke, and patients with end-stage renal disease have an 84% higher rate of suicide compared to the U.S. population. Patients with kidney transplants have a 75% highe rrate of suicide compared to national rates. Patients with self-reported kidney disease attempt suicide at three times the rate of the U.S. population. Depression among dialysis patients may adversely affect mortality, possibly independent of dialysis adequacy. The risk of suicide is highest in the first 3 months of dialysis, which suggests that suicideis prompted by failure to cope with stress rather than declining health status. With drawal from dialysis before death is common and occurs approximately 100 times more often than suicide. Studies suggest poorer survival among depressed dialysis patients. Psychiatric illness is common among patients with chronic medical disorders, particularly in
those with end-stage renal disease (ESRD): Patients with end-stage renal disease have a high prevalence of depression, especially at dialysis initiation. In renal patients, exclude uremia and assure adequate dialysis before diagnosing depression; since symptoms of depression and those originating from a metabolic disorder are similar: under-dialysis, including anorexia and failure to thrive, may mirror depression; correction of fluctuating blood pressure, nausea, or other gastrointestinal complaints may improve quality of life and may effectively treat psychosocial markers suggesting depression; common complaints in the dialysis patient, such as chronic fatigue, weakness, and constipation, may reflect a psychosocial disturbance. There quest to with draw from dialysis may suggest depression. In patients who appearto have intact decision-making capacity, exclude the subtle presence of factors such as situational depression, mild dementia, and uremic or toxic encephalopathy. The diagnosis of depression can be confounded by patients who do not realize - or deny - they are clinically depressed due to suicidal intent or the stigma associated with mental
illness.

86
Q

Why are potential problems sometimes missed in medical practice?


A
  1. Physicians receive inadequate training. You would not learn CPR at your first cardiac arrest. You need to learn the principles of emergency psychiatry before a suicidal patient comes into your office. Unfortunately, most residencies spend very little, if any, time on psychiatric matters although surgeons will see a lot of trauma; family physicians, internists and pediatricians see the whole panoply of abuse, incest, suicide and so forth. 2. Same-class bias and other counter-transference issues may prevent you from asking patients - especially those who you feel you are like - tough questions about lethality (suicide, homicide, child and spouse abuse, incest, and drug abuse). It is said that physicians will diagnose alcoholism only in those who drink more than they do. 3. Inadequate mental health backup. Physicians are less likely to identify tough psychiatric problems in their patients unless they feel they have someone to back them up and to whom they can send their patients.
87
Q

DANGEROUS BEHAVIORS = MALADAPTIVE RESPONSE TO STRESS

A

Psychiatric emergencies may be thought of as resulting from maladaptive responses to stress. For example, a stressor (e.g. loss) leads first to denial then may precipitate disorganization and symptom formation, then maladaptive responses (e.g. suicide, abuse) and then re-equilibration. Even normal life events can precipitate dangerous behaviors in susceptible persons. For example, pregnancy is a high-risk time for spousal violence and incest in susceptible men. Patients come – or are brought – to us only in the phases of Disorganization /Symptom Formation or when they have resorted to a maladaptive behavior such as suicide or abuse. According to this model, a vulnerable patient is first stressed and then resorts to dangerous behaviors. The healthier the patient, the more profound the stress has to be to precipitate problems. The KEY question for us is to figure out what the patient is trying to accomplish and fix, albeit in this destructive way, when they resort to abuse, suicide and so forth. For example, suicide may be a way of reuniting with someone who is dead or punishing someone who is alive. Finally, according to this model, the period of highest lethality typically does not last long. Either the problem is solved – e.g. the spouse returns – or the patient makes a suicide attempt. This fact, that the period of highest lethality is short, is why brief hospitalizations can be life-saving.

88
Q

COMMON PRESENTATIONS OF PSYCHIATRIC EMERGENCIES IN MEDICAL PRACTICE

A

None are pathognomonic and the only way to really find out what is going on with your patients is to ASK. HOW YOU ASK IS LESS IMPORTANT THAN THAT YOU ASK. Patients with vague somatic complaints, aches, and pains or multiple visits to physicians: patients may be labeled as crocks, WADAOs (weak and dizzy all over), hypochondriacs, somatizers, etc., which is fine. You can make such pejorative diagnoses to yourself; what really matters is what you do next, how you evaluate and treat these people. Symptoms and signs of depression, anxiety, drug and alcohol abuse e.g. insomnia or hypersomnia, fatigue, worry, weight gain or loss, irritability, difficulty with concentration, problems at home, work or school. Remember that studies have found that 70% of depressions are missed in medical practice. Accidents, multiple or unexplained trauma (child/spouse abuse), GU complaints or early pregnancy (incest), GI complaints (alcoholism), problem pregnancies (abuse), school problems (child abuse, incest) and so forth. Symptoms and signs of family disruption, school problems, truancy all may be indications of spousal violence, child abuse or incest. Sexual promiscuity, running away and suicidal activity is known as the “incest triad” and indicates a very high risk of incest. One psychiatric emergency may mask another. Emergencies come in bunches and often overlap. For instance, alcoholism may coexist with abuse or suicide; spousal violence may co- exist with child abuse. Therefore, if you find one, look for the others.

89
Q

The neurochemistry of violence

A

Serotonin has been implicated in the control of violent behaviors – both suicide and violence directed towards others. There is a remarkably consistent association between low cerebrospinal fluid concentrations of the Serotonin metabolite 5- HIAA and suicidal behavior. This association is found not only in depressive disorders but has been found in schizophrenia, some personality disorders, and in certain impulse disorders but not in bipolar disorder. Low concentrations of CSF 5-HIAA is associated with a real increase in suicide risk. Studies in violent criminals in non-human primates suggest that aggression dyscontrol may partly explain this association. There is a lot of evidence that supports the association between aggression and alcohol consumption as well. Alcohol consumption has a major effect on Serotonin metabolism. The “Serotonin deficiency hypothesis” of alcohol-induced aggressive behavior postulates that people susceptible to aggression after consuming alcohol exhibit a marked depletion of their brain Serotonin. This makes them prone to aggressive outbursts in response to environmental or psychological stimuli or situations. In addition, a significant association has been found between low cholesterol levels and violence – towards self and others – across many studies. It is postulated that lowered cholesterol levels may lead to lowered brain Serotonin activity; this may in turn, lead to increased violence.

90
Q

Suicide Assessment: changeable states and immutable traits

A
  1. The Patient’s History and Demographics: traits. History of suicide attempt or impulsive, violent behavior? Family history of suicide, male, Caucasian, single, recent loss, divorced, widowed? Chronic illness? History of major depression, schizophrenia, bipolar mood disorder, drug or alcohol problems, cognitive disorder, anorexia nervosa, personality disorder with axis I co-morbidity all push people into a higher risk category. In people with Anorexia Nervosa, for instance, suicide occurs in 5-10% of patients within 10 years and almost 20% of patients within 20 years of onset of the disease. Suicide also occurs in approximately 15% of patients diagnosed with alcohol abuse or dependency, patients with depression, and patients diagnosed as schizophrenic. 2. The Acute Clinical Picture: changeable state. Passive death wishes? Active suicide desire? Death wishes in family and friends? Hopelessness and lack of future plans, lethal suicide plan, suicide note, intoxicated, psychotic, weapons, etc…
91
Q

the work up of sucide

A

Developing a differential depends on your “snapshot” of the patient now and your assessment of your patient’s symptoms over time, i.e. the clinical course of your patient.
I structure my work-up around the following Seven Diagnostic Questions. Information about these questions should be gleaned from as many sources as possible:Why is the patient here now? What does the patient want and expect to accomplish? Is a general medical illness contributing to the patient’s difficulties? How lethal – suicide, abuse, incest, violence – do you judge the situation to be? Are family and friends part of the problem or part of the solution? What are the patient’s cultural expectations/explanations/treatments for their illness? What is the patient’s psychiatric diagnosis? Lethality and medical issues must be answered first and constitute the core of the first-line of the emergency evaluation.

92
Q

ERRORS MADE IN THE EVALUATION OF SUICIDE IN PSYCHIATRIC PATIENTS

A

Not corroborating the patient’s story with family and friends. This is the number one mistake made by psychiatrists and non-psychiatrists alike. Failing to review old records. Failing to seek consultation: collegial, medical or psychiatric. Share your worry about tough patients and do not assume that you have to know and do everything. Avoiding specific questions about guns, plans, fantasies and so forth. Ignoring risk enhancing factors such as the presence of a psychotic disorder (i.e. command hallucinations, paranoia), drug/alcohol abuse, feelings of hopelessness, a prior history of abuse, suicide, etc. Such factors always raise the risks of lethality.

93
Q

TREATMENT Of Suicidal ideation

A

Treatment clearly depends on your diagnostic assessment of dangerousness but it is better
safe than sorry. Remember that you can hold a patient against their will if you judge them mentally ill and
to be a potential harm to themselves and others or gravely disabled. You also have a duty to warn potential victims as well as care for your patient. You have a duty to report suspected child abuse and incest as well as domestic violence.

94
Q

Before Sending a Patient Home from suicide watch

A

Following the evaluation of the specific emergency (at the very least lethality and medical issues), there are four issues the practitioner might examine in order to judge the adequacy of the immediate treatment plan. Do I understand the seriousness of the patient’s problem or am I going along with the patient’s or family’s denial? Two notions have helped me: A false alarm is better than no alarm at all and the last person to see the patient gets to make the last mistake. What was the patient attempting to accomplish with their destructive behavior and did they accomplish it? In other words, has the crisis passed or is the patient still overwhelmed with whatever stressed them in the first place? Do I dare let the patient go home or is the patient going back into the pathogenic environment which contributed to his/her difficulties in the first place? Will the patient follow the treatment plan? What contingency plans have I made if the patient does not follow through? Do I know how to reach the patient and have the correct address and telephone number? Do I have a friend, relative or neighbor I can call? A good follow-up plan – and close follow-up makes up for any mistakes in clinical judgment.

95
Q

Normal plasma value of Na

A

140 ± 3 mEq/L (Tells you about the relative amount of water in the ECF compared with Na. It tells you nothing about total body Na; best considered as an indirect but readily available assessment of plasma osmolality that is accurate under most (but not all) circumstances).

96
Q

Normal plasma value of K

A

4.5 ± 0.6 mEq/L (Tells you about plasma K; relatively poor indicator of total body K).

97
Q

Normal plasma value of Cl

A

104 ± 3 mEq/L (Generally considered a passive anion; used in the anion gap calculation as described below).

98
Q

Normal plasma value of Total CO2 (tCO2)

A

27 ± 2 mEq/L (Total CO2 content of blood; about 3 mEq/L higher than the arterial HCO3- because of dissolved CO2. Used for calculation of anion gap).

99
Q

Normal plasma value of Glucose (Fasting)

A

90 ± 30 mg/dL

100
Q

Normal plasma value of Creatinine

A

1.0 ± 0.3 mg/dL (Used to estimate renal function; reciprocal (1/cr)
directly proportional to CrCl and, indirectly, to GFR).

101
Q

Normal plasma value of BUN

A

12 ± 4 mg/dL

102
Q

Normal plasma value of Phosphorus

A

4.0 ± 1.0 mg/dL

103
Q

Normal plasma value of Calcium

A

9.5 ± 1.0 mg/dL

104
Q

Normal plasma value of Cholesterol

A

140-200 mg/dL

105
Q

Normal plasma value of Osmolality

A

285 ± 3 mosm/kg H2O (Direct measurement of plasma osmolality;
difficult to obtain in a short time from a clinical lab).

106
Q

Normal plasma value of Plasma Anion Gap

A

Na–(Cl+tCO2)=9±2mEq/L

107
Q

Normal urine value of Na

A

low suggests avid tubular sodium reabsorption ( 40 mEq/L)

108
Q

Normal urine value of K

A

must be considered in light of plasma potassium.

109
Q

Normal urine value of Creatinine

A

usually viewed in concert with plasma creatinine; a UCr/PCr value
greater than 20 suggests avid tubular water reabsorption, a value less than 10
suggests less avid water reabsorption.

110
Q

Normal urine value of Osmolality

A

(again, no “normal range) 285 is isotonic -normally can dilute to less
than 50-100 mosm/kg H2O, -normally can concentrate to greater than 1000 mosm/
kg H2O

111
Q

Normal urine value of Specific gravity

A

1.010 is isotonic ( this is concentrated)

112
Q

Normal Blood Gas Value of pH

A

7.42 ± 0.02

113
Q

Normal Blood Gas Value of pO2

A

94 + 8 (remember, we’re assuming sea level)

114
Q

Normal Blood Gas Value of pCO2

A

38 ± 2 (remember, we’re assuming sea level)

115
Q

Normal Blood Gas Value of HCO3-

A

24 ± 2 (remember, we’re assuming sea level). Note that both venous tCO2
and arterial HCO3- are calculated from the Henderson-Hasselbach equation and are equally accurate.

116
Q

Acute kidney injury (AKI)

A

defined as a rapid reduction in glomerular filtration rate manifested by a rise in plasma creatinine (Pcr) concentration, urea and other nitrogenous waste products. AKI results in reduced clearance of nitrogenous waste products to produce a state called azotemia (Azote is the French word for nitrogen, hence azote-emia or increased nitrogen in blood). Three broad types of disorders can cause AKI: Pre-renal acotemia, post renal axotemia or obstructive neuropathy, or intrinsic renal disease

117
Q

Pre-renal azotemia

A

a decrease in GFR due to decreases in renal plasma flow and/or renal perfusion pressure.

118
Q

Post-renal azotemia or obstructive nephropathy

A

a decrease in GFR due to obstruction of urine flow.

119
Q

Intrinsic renal disease

A

a decrease in GFR due to direct injury to the kidneys (may be due to a variety of insults).
Uremia refers to the constellation of signs and symptoms of multiple organ dysfunction caused by retention of “uremic toxins” and lack of renal hormones due to acute or chronic kidney injury. Such symptoms include nausea, vomiting, abdominal pain, diarrhea, weakness and fatigue.

120
Q

Azotemia

A

Buildup of nitrogenous wastes in the blood; i.e. Blood Urea Nitrogen
(BUN) and serum creatinine are increased.

121
Q

Prerenal azotemia

A

Normal renal function is dependent on adequate renal perfusion. The kidneys receive up to 25% of the cardiac output resulting in more than 1 liter of blood flow per minute. This high rate of renal blood flow (RBF) is required not only to maintain GFR, but also to maintain renal oxygen supply required for ion transport. A substantial reduction in renal perfusion may be sufficient to diminish effective filtration pressure and thus lower GFR. Prerenal azotemia is the most common cause of an abrupt fall in GFR in a hospitalized patient. In general, prompt restoration of intravascular volume restores RBF and GFR and prevents structural ischemic renal injury. Prolonged pre-renal azotemia, however, may result in acute tubular necrosis. Be careful not to associate pre-renal azotemia with only hypovolemia because there are certain hypervolemic states that can cause a pre-renal picture, e.g. congestive heart failure (CHF) and cirrhosis. People with CHF have poor cardiac output and therefore reduced blood flow to the kidneys. If CHF is severe enough, blood ‘backs-up’ into the pulmonary circuit resulting in pulmonary edema and a hypervolemic state. CHF and cirrhosis are characterized by a low effective arterial blood volume (EABV) and reduced renal perfusion. Therefore, these conditions are classified as pre-renal even though they may cause hypervolemia. The renal tubules function normally in prerenal azotemia and will concentrate the urine and reabsorb sodium avidly. Thus, urine sodium concentration will be low (less than 20 mEq/L) and urine creatinine concentration will be high (Ucr/Pcr ratio > 20).

122
Q

The ratio of the clearance of sodium to creatinine (also called the fractional excretion of sodium or FENa)

A

is calculated as follows: FENa = (UNa/PNa) ÷ (UCr/Pcr ) X 100 (expressed in %) This equation takes both of these features into account and is useful in separating prerenal azotemia from other causes of AKI. In general, the FENa is 2% when AKI is caused by other pathologies. There are exceptions to this rule, i.e. radiocontrast and rhabdomyolysis (severe muscle breakdown) can cause FENa

123
Q

Postrenal azotemia (obstruction)

A

Obstruction to urine flow is another potentially treatable form of AKI. With urinary tract obstruction, increases in intratubular pressure typically cause the low GFR acutely but if obstruction is prolonged, an intense renal vasoconstriction usually develops and results in a persistent decrease in GFR. Except in patients with single kidneys or with pre-existing bilateral parenchymal disease and reduction in renal function, obstruction must be bilateral to produce significant kidney injury. Obstructive uropathy will be encountered in 2 to 15% of all patients with AKI. Obstructive uropathy often presents as no urine flow (anuria) or intermittent urine flow, but may also present as nonoliguric AKI. Obstruction is the most common cause of anuria.

124
Q

Causes of Postrenal Azotemia

A

Obstruction of ureters: Extraureteral (e.g. carcinoma of the cervix, endometriosis, retroperitoneal fibrosis, ureteral ligation). Intraureteral (e.g. stones, blood clots, sloughed papilla). Bladder outlet obstruction (e.g. bladder carcinoma, urinary infection, neuropathy). Urethral obstruction (e.g. posterior urethral valves, prostatic hypertrophy or carcinoma).

125
Q

Urinary obstruction

A

generally results in derangement of tubular function that affects sodium and water handling. In general there is an impairment of tubular sodium reabsorption that is reflected by high urinary sodium concentrations (> 40 mEq/L) and an impairment of water reabsorption that results in low urine creatinine concentrations (Ucr/Pcr, ratio 2% in urinary obstruction.

126
Q

The most sensitive test for the diagnosis of obstruction

A

the renal ultrasound evaluation (US). An US will demonstrate obstruction as an expansion of the collecting system (hydronephrosis) in as many as 98% of cases. For obstruction at the level of the urethra, placement of a bladder catheter following voiding may confirm the diagnosis. If there is still considerable urine present, obstruction to flow of this urine has occurred. This volume of urine is called the ‘post-void residual’. A bladder catheter will also bypass the obstruction and provide treatment. Prompt relief of acute obstruction is usually associated with complete return of renal function, but prolonged obstruction is often accompanied by incomplete return of renal function after relief of the obstruction.

127
Q

Intrinsic renal disease

A

Intrinsic renal disease (i.e. disease that affects the renal parenchyma) is divided into four types based on the four structures found in the kidney: vessels, glomeruli, interstitium and tubules. The most commom injury is acute tubular necrosis (ATN) that is caused by either ischemia or nephrotoxins (which are typically medications). It should be noted that the term ATN may be somewhat of a misnomer as histologic studies have not consistently demonstrated tubular necrosis.

128
Q

Intrinsic Renal Diseases Causing AKI

A

Vascular diseases: e.g. cholesterol emboli, renal vein thrombosis. Glomerular diseases: e.g. acute glomerulonephritis, hemolytic uremic
syndrome

129
Q

Interstitial diseases

A

Acute interstitial nephritis (e.g. allergic interstitial

nephritis (AIN)), infection, myeloma kidney.

130
Q

Tubular diseases

A

Ischemic or nephrotoxic acute tubular necrosis (ATN).

131
Q

Mechanisms of decreased gfr in atn

A

The pathophysiology of human ATN is still uncertain, although considerable work has been performed on this topic. In experimental ATN, it appears that different physiologic factors may mediate the initiation and maintenance of the decreases in GFR that is the hallmark of ATN. The physiologic factors are usually subdivided into vascular and tubular. The vascular factors include decreases in renal blood flow and decreases in glomerular permeability (Kf), whereas the tubular factors include tubular obstruction (by cellular debris) and backleak of glomerular filtrate (across an incompetent tubular basement membrane). Animal data suggests that most ischemic human ATN involves a reduction in renal blood flow as the primary event. This causes ischemia and consequently, injury to the proximal tubular epithelial cells. The injured (or sometimes necrotic) cells break away from their tubular basement membrane exposing bare sections of tubule through which ‘backleak’ is thought to occur. The cells clog up the proximal tubule causing tubular obstruction and maintain the low GFR. More recent studies addressing renal vascular reactivity suggest that ischemic injury may also produce a failure of autoregulation of renal blood flow. This may contribute to the maintenance of ATN allowing recurrent ischemic insults to result from relatively mild episodes of systemic hypotension. Patients with ATN may be oliguric or nonoliguric.

132
Q

Oliguric vs Nonoliguric patients with ATN

A

Nonoliguric patients have a lower mortality rate (about 20% as compared to 60-80% for oliguric ATN) perhaps because: Nonoliguric ATN is a less severe injury or Management of fluid and electrolyte status is easier.
Infections due to decreased leukocyte function and gastrointestinal tract hemorrhage due to increased acid secretion (stress, ulcers) are two serious complications associated with ATN. Ischemic AKI with ATN is typically characterized by an oliguric phase of about 7- 14 days, followed by a nonoliguric phase of 10-14 days. However, this is not always the rule in clinical practice where prolonged oliguric phases as well as initial nonoliguric presentations are both common.

133
Q

History and physical examination of AKI

A

Look for evidence that will help you decide if your patient has pre-, renal, or post-renal AKI. Some important signs and symptoms are: Intravascular volume depletion is suggested by a decrease in weight, flat neck veins, and postural changes in blood pressure and/or pulse. The presence of these signs and symptoms would suggest a pre-renal etiology of AKI. Cardiac dysfunction is suggested by edema, pulmonary rales, and a S3 gallop. Again the presence of these signs and symptoms would suggest a pre-renal etiology of AKI. Note that in these instances, the patients demonstrate signs of hypervolemia (see section on Pre-renal Azotemia). A history of exposure to renal insults associated with ATN, i.e. hypotension, surgery involving large blood loss, transfusion reactions, or exposure to radiocontrast dye used for CT scans and cardiac catheterization. Evidence of urinary obstruction e.g. anuria, intermittent anuria or large swings in urine flow rate. Evidence of other causes of AKI, e.g. rash and fever after exposure to ampicillin (antibiotic) suggests AIN (allergic interstitial nephritis)

134
Q

Urinalysis and sediment examination of AKI

A

This represents an important extension of the basic History and Physical, and must be performed on all patients with AKI. The urinalysis provides useful information including the tonicity of the urine as well as the presence or absence of heme pigments. For example, high tonicity occurs when the kidney retains water due to increased antidiuretic hormone (ADH). These include pre- renal states and SIADH (more in the hyponatremia and water chapter). The presence of heme pigments (detected by a positive blood test on the urine dipstick) without microscopic evidence of RBCs in the urine would suggests rhabdomyolysis or hemolysis. Rhabdomyolysis is a syndrome characterized by muscle necrosis and release of intracellular muscle constituents (including myoglobin) into the circulation. It is a common cause of AKI, especially in trauma patients.

135
Q

Three main components of urine analysis

A

Macroscopic or gross exam, Dipstick chemical analysis, and Microscopic exam

136
Q

Macroscopic exam

A

This is a direct, visual observation of collected urine. Fresh urine from a normal patient is clear and pale to dark yellow in color. Patients who are ill may have cloudy or turbid urine. RBCs in the urine may cause the color to appear red or reddish brown (“cola colored”).

137
Q

Dipstick analysis

A

Dipsticks are long thin strips of inert plastic that have square reaction pads that provide color readouts based on biochemical reactions with the urine. Reaction pads include: glucose, protein, heme, bilirubin, ketones, and leukocyte (esterase and nitrite).

138
Q

Microscopic exam

A
This is for identifying formed elements. These include cells, casts, and crystals. Cells include WBCs, RBCs, bacterial and epithelial cells. RBCs in the urine can 
have a variety of shapes and appearances depending on their origins within the 
genitourinary system (glomerular versus non-glomerular origin).  The renal tubules secrete mucoproteins and when these proteins “gel” in the 
tubules, they form casts. (see figure 6) They are cylindrical in shape, just like the lumen of a tubule. Anything present in the tubules when these casts are formed will be trapped in the casts. Hyaline casts have no cells within them and they are normal findings in healthy individuals. RBC and WBC casts reflect pathology since RBCs and WBCs are normally not in the tubules. They are associated with glomerulonephritis and AIN, respectively. To perform a microscopic exam, you centrifuge ten to 15 mL of fresh urine at 400-450 g for 5 minutes (procedure may vary). The supernatant urine is decanted and the sediment is resuspended in a small volume of remaining urine. You then place a drop on a slide for microscopic examination.
139
Q

Urinary casts

A

are typically formed in the distal convoluted tubule (DCT) or the collecting duct (distal nephron). The proximal convoluted tubule (PCT) and loop of Henle are not locations for cast formation. Hyaline casts are composed primarily of a mucoprotein (Tamm-Horsfall protein) secreted by tubule cells. The Tamm-Horsfall protein secretion (dots) is illustrated in the diagram, forming a hyaline cast in the collecting duct.

140
Q

Prerenal azotemia UA Pattern

A

Relatively high specific gravity, no heme pigment, normal sediment (i.e. any casts are waxy or finely granular).

141
Q

Glomerulonephritis UA Pattern

A

Variable tonicity, + heme pigment, sediment exam reveals RBC and RBC casts.

142
Q

AIN

A

Isotonic urine, +/- heme pigment, white blood cell casts, eosinophils (with allergic interstitial nephritis)

143
Q

Vascular UA Pattern

A

Variable tonicity, ± hematuria

144
Q

ATN UA Pattern

A

Typically isotonic, variable heme pigment (+ if from hemolysis or rhabdomyolysis). Sediment exam will show pigmented coarsely granular casts and renal tubular epithelial cells (RTEs).

145
Q

Obstruction UA Pattern

A

Tonicity usually isotonic or hypotonic, usually heme is negative unless superimposed infection. Micro may be totally benign or show evidence of superimposed infection (e.g. RBCs & WBCs).

146
Q

Urine Chemistries

A

The pathology of the renal injury may be reflected in the biochemical content of the urine. In general, a low FENa suggests prerenal azotemia. A high FENa is associated with other causes of acute kidney injury.
Exceptions to this rule include: Nonoliguric ATN: 20% of such cases may have a low FENa.
ATN due to radiocontrast exposure during CT scans and cardiac catheterization. Radiocontrast agents cause vasoconstriction and ATN. Early in the course of AKI, the vasoconstriction may predominate and cause a low FENa.
ATN due to pigment exposure as is seen during rhabdomyolysis or hemolysis. Rhabdomyolysis is a condition where a low FENa may occur even with oliguric A TN.

147
Q

COMPLICATIONS OF ACUTE KIDNEY INJURY

A

Kidney failure results in loss of fluid balance and electrolyte regulation. AKI is therefore often complicated by volume overload (e.g. pulmonary edema) and electrolyte abnormalities (e.g. hyperkalemia and acidosis). A patient with AKI may also develop signs and symptoms related to uremia (discussed in the chapter, Chronic Kidney injury). Mortality in patients with ATN is still extremely high (approaching 60%). Common causes of death in these patients include infections and gastrointestinal bleeding.

148
Q

THERAPY OF ACUTE KIDNEY INJURY

A

Therapy of AKI is based on the cause of the renal insult. Prerenal azotemia is treated by optimizing renal perfusion. This may involve improving cardiac output in CHF, or replacing intravascular volume during hypovolemia. Post-renal azotemia is treated by relieving the obstruction. Therapy of ATN is best addressed before ATN develops, i.e. by avoiding risk factors that predispose to ATN, such as pre-renal azotemia or nephrotoxins. If ATN does develop, fluid overload and electrolyte disturbances are usually treated medically at first. You will learn about these medical treatments in the ensuing chapters. If these measures are insufficient, the patient is treated with procedures known as renal replacement therapy. These therapies involve a procedure called dialysis in which fluid, electrolytes, and nitrogenous wastes are removed from plasma by external devices. Generally, when ATN is oliguric, an attempt is made to convert it to a nonoliguric form using loop diuretics although it has not been clearly demonstrated that this improves morbidity or mortality.

149
Q

Acute Renal Failure versus Acute Kidney Injury

A

There has been a recent change in terminology from “acute renal failure” to “acute kidney injury.” For the purpose of this course, we will refer to acute renal failure as “acute kidney injury.” There are several reasons for this change. The use of “kidney” rather than “renal” parallels the change in terminology from chronic kidney injury to chronic kidney disease. Very simply, the lay population may know what a kidney is but have no idea what “renal” means. The change from “failure” to “injury” recognizes the fact that many patients with acute renal dysfunction are not in kidney failure. The new terminology also includes graded definitions of renal injury, known as the RIFLE criteria (Risk, Injury, Failure, Loss, End-Stage). Having such a classification will help with gauging the severity of acute kidney injuries, as not all injuries are alike and thus, not all kidney injuries should be treated in the same fashion.

150
Q

Glomerular Anatomy

A

Glomeruli are the filtering units of the kidney. About 20% of the cardiac output goes to the kidney and glomeruli. The glomerulus is illustrated schematically in Fig 1 and shows 3 normal glomerular capillary loops. The glomerular capillary wall is uniquely permeable to salt, water and metabolic waste products such as creatinine and urea. About 150 liters of glomerular filtrate containing these materials in concentrations equivalent to those in plasma enter the renal tubule daily. Tubular reabsorptive processes then modify the glomerular filtrate as it passes down the nephron resulting in excretion of 1 to 2 liters of final urine. However, larger materials such as cells and proteins are not normally filtered at the glomerulus. The glomerular filtration barrier consists of the endothelial cell layer, the glomerular basement membrane (GBM) itself and the glomerular epithelial cells, which abut on the GBM as foot processes connected by, slit diaphragms. The glomerulus is essentially a “ball of capillaries” which contains three major cell populations. The glomerular endothelial cell lines the inner side of the GBM, and is characterized by fenestrations (gaps of 100mm), which allow direct access of plasma components to the GBM. The glomerular mesangial cell is a centrally located cell that, a) secretes a basement membrane-like matrix which acts as a structural support to the glomerulus, b) has smooth muscle-like properties and can contract, thus affecting capillary surface area and filtration, and c) has some macrophage-like properties including the ability to secrete cytokines, growth factors, proteases and oxidants. The glomerular basement membrane (GBM) is composed mainly of type IV collagen that acts as the “backbone” of GBM; laminin and entactin, glycoproteins important in endothelial and epithelial cell attachment, and heparan sulfate proteoglycan which is important in providing a negative charge to the GBM. The most important barrier to protein is the filtration slit diaphragm that extends between The glomerular epithelial cell (also called podocyte) rests on the outside of the GBM and interdigitates with the GBM via long “foot processes” which are separated from each other by thin slit-Iike diaphragms

151
Q

Glomerular Capillary Is Normally Impermeable to Proteins: Mechanism

A

Normally, the glomerular capillary wall acts as a highly selective filter, allowing for the filtration of water, electrolytes, and small molecular weight solutes. However, it is a barrier to large molecular weight (>100 kd or >42 angstroms) or negatively charged molecules (i.e., most proteins). The ‘charge’ barrier is due to the fact that the basement membrane is highly negatively charged (due to the presence of heparan sulfates) and the endothelium and podocyte are also coated in negatively charged proteins (consisting primarily of sialoproteins). This leads to a “charge-charge repulsion” with most proteins (since most proteins are also negatively charged). The most important mechanism involves a “size barrier”. While the GBM provides some protection, most studies show that the critical site is the slit pore diaphragm that extends between the podocyte foot processes. Recent studies have shown that this diaphragm consists of interlocking proteins that form a zipper with intervening pores that are just small enough to prevent most proteins from escaping through into Bowman’s space. The primary protein in the slit diaphragm has recently been identified and is termed nephrin. A mutation in the nephrin gene is responsible for the congenital nephrotic syndrome of the Finish type.

152
Q

Normal Protein Excretion

A

While the slit diaphragm is an effective barrier to protein filtration, normally about 500 mg of albumin escapes into Bowman’s space. However, most of this albumin is reabsorbed by the proximal tubule where it is degraded, and the normal excretion of albumin is less than 20 mg/day. Other proteins are also excreted normally in the urinary tract; these consist of the Tamm-Horsfall protein (from the medullary thick ascending limb or mTAL cell) and IgA; these latter proteins may help protect against bacterial infections.

153
Q

Abnormal Protein Excretion

A

Albumin greater than 30 mg/day is abnormal. When it is in the range of 30 to 300 mg/day, it is generally not detectable by routine urinalyses; this is called microalbuminuria. Persistent microalbuminuria is strongly suggestive of early glomerular damage and often determined in diabetic subjects as it can be used to predict the development of diabetic nephropathy. Albumin > 300 mg per day can be identified in urinalyses as a positive dipstick (range 0-4+). Albumin between 300 mg and 1 to 2 gm/day may be due to either glomerular or tubular disease. However, albuminuria > 3 gm/d is only observed if there is a defect in glomerular permeability. When the albumin excretion is > 3 to 3.5 g/d, it is usually severe enough to result in a decrease in the serum albumin, often with the development of edema. This is termed nephrotic range-proteinuria, and the clinical syndrome as Nephrotic syndrome. Low grade proteinuria (albumin excretion of 20 gms) that can be detected by 24 hour urine protein measurements yet be associated with a negative urinalysis dipstick which only detects albumin.

154
Q

Nephrotic syndrome-Definition

A

Glomerular diseases generally present as one of two clinical syndromes, i.e, the nephrotic syndrome, in which the major glomerular abnormality is an excessive leak of protein through the glomerular capillary wall into the urinary space, and the nephritic syndrome, in which active inflammation within the glomerulus leads to damage to the glomerulus with subsequent loss of filtration and a reduction in the GFR. The Nephrotic syndrome results as a consequence of marked albuminuria (>3.5g/d), which leads to hypoalbuminemia and edema. It is defined as the following:Proteinuria (>3.5 gm/day or >40 mg/hr/M2 in children), Hypoalbuminemia (

155
Q

Nephrotic syndrome-pathophysiology

A

signs include proteinuria, hypoalbuminemia, edema, hyperlipidemia, and lipiduria

156
Q

Proteinuria in nephrotic syndrome

A

Nephrotic range proteinuria almost always means that there is a disruption of the slit diaphragm. This may result from a specific defect of the diaphragm (such as a mutation of nephrin) or if the podocyte is injured. I

157
Q

Hvpoalbuminemia in nephrotic syndrome

A

results from the proteinuria and increased catabolism of reabsorbed protein in the renal tubules. Protein synthesis by the liver is increased but can not compensate completely for the urinary losses.

158
Q

Edema in nephrotic syndrome

A

results from two mechanisms. The classic mechanism is mediated by a decrease in serum albumin which decreases plasma oncotic pressure, resulting in filtration of fluid into the interstitial space on the basis of Starling’s law, leading to a decrease in intravascular volume with stimulation of renin-angjotensin-aldosterone and vasopressin leading to additional salt and water retention, respectively. This is observed primarily in children with nephrotic syndrome. Most adults, however, do not show evidence of volume depletion. In these patients there is a primary renal defect in sodium excretion, resulting in volume expansion, and fluid movement into the interstitium due to both the low oncotic pressure and high hydrostatic pressure.

159
Q

Hyperlipidemia in nephrotic syndrome

A

with an increase in serum cholesterol results from increased lipoprotein synthesis (­ VLDL, ­ LDL) in the liver and decreased peripheral removal of VLDL. The stimulus for increased lipoprotein synthesis seems to be the decrease in plasma oncotic pressure. The risk for coronary artery disease/atherosclerosis relates to the duration of the nephrotic syndrome, the presence of hypertension, and whether the elevated cholesterol is associated with a low HDL fraction.

160
Q

Lipiduria

A

results from the combination of increased capillary wall permeability and the hyperlipidemja, and results in the presence of fat (i.e., oval fat bodies, free fat or fatty casts) in the urine which have the appearance of a “Maltese cross” under polarized light.

161
Q

Additional clinical features are common in nephrotic syndrome

A

increase risk for infectios, increased risk for thrombosis, poor growth in children and osteomalacia, and protein malnutrition.

162
Q

Increased risk for infections in nephrotic syndrome

A

especially pneumonia and spontaneous bacterial peritonitis in children – usuaIly due to pneumococcus as a consequence of urinary loss of IgG and complement ( esp. factor B) in the urine. Management. includes vaccine, pneumococcal vaccine, IgG therapy for recurrent infections.

163
Q

lncreased risk for thrombosis in nephrotic syndrome

A

due to an increase in synthesis of coagulation factors (fibrinogen, factors V, VIII, IX, X) by the liver, urinary loss of anti-thrombin III, increased platelet aggregability, and impairment of the fibrinolytic system. This may presents as lower extremity thrombosis of the veins, or sometimes as renal vein thrombosis. Rarely arterial thromboses may occur. 
Acute renal vein thrombosis may result in flank pain, an enlarged kidney by ultrasound, and hematuria. Chronic renal vein thrombosis is often asymptomatic due to the time for the development of collateral circulation. Both may be associated with pulmonary emboli. Management requires chronic anti-coagulation.

164
Q

Poor growth in children and osteomalacia in nephrotic syndrome

A

may occur due to loss of vitamin D and its binding protein in the urine, and can be detected by measuring ionized calcium and 25-0H vitamin D levels.

165
Q

Protein malnutrition in nephrotic syndrome

A

may occur if hypoalbuminemia is severe.

166
Q

Classification of Glomerular Disease

A

Glomerular disease can present as nephrotic syndrome, nephritic syndrome, asymptomatic proteinuria, or asymptomatic hematuria. Nephrotic syndrome presents with the pentad of severe proteinuria (Uprot>3.5g/d), hypoalbuminemia (serum albumin 250 mg/dl), pitting edema, and lipiduria. Nephritic syndrome presents with micro hematuria and occasionally red cell casts, non-nephrotic proteinuria, decreased GFR, hypertension and edema. Asymptomatic proteinuria may have either a glomerular or tubular pathogenic mechanism. Glomerular diseases (particularly IgA disease) may present as micro- or macro hematuria in the absence of proteinuria, but workup should include IVP, cystoscopy, urinary calcium/oxalate measurements, and culture to rule out tumor, infection, congenital defects, and kidney stones. Trauma may also result in hematuria.

167
Q

Major causes of Idiopathic Nephrotic syndrome

A

Nephrotic syndrome reflects injury to the podocyte with the massive transit of albuminuria. It may be due to primary diseases of the glomerulus (termed idiopathic nephrotic syndrome) or due to systemic diseases that affect the glomerulus. The most common secondary causes are diabetes mellitus, systemic lupus (the membranous type of presentation), light chain deposition (a type of plasma cell dyscrasia that may be associated with multiple myeloma) and amyloidosis (also occasionally associated with multiple myeloma). Idiopathic nephrotic syndrome can have many causes. The four major histologic patterns are minimal change disease, idiopathic focal and segmental glomerulosclerosis (FSGS), membranous nephropathy (MN), and membranoproliferative glomerulonephritis (MPGN).

168
Q

Definition of Minimal change disease

A

Minimal change disease is a common cause of nephrotic syndrome. The name reflects the fact that the glomeruli appear normal by light microscopy.

169
Q

Epidemiology of Minimal change disease

A

Minimal change disease is the most common cause of idiopathic nephrotic syndrome in children and remains a significant cause in adults. It tends to affect relatively young children with a peak incidence in the 2-4 year age range. In children it is twice as common in males as in females whereas in adults the ratio is 1.1. A second peak occurs in the 5th or 6th decade.

170
Q

Pathology of Minimal change disease

A

Light microscopy is normal; by immunofluorescence there is no immunoglobulin or complement; by electron microscopy there is only foot process fusion, which is observed in all forms of nephrotic syndrome.

171
Q

Etiology of Minimal change disease

A

The etiology of minimal change disease is unknown but may involve a T cell derived circulating permeability factor that acts directly on the glomerulus to damage the podocye and the permeability barrier. Evidence for a circulating factor includes the observation that the disease develops in normal kidneys immediately following transplant; and diseased kidneys become normal following transplant. Evidence that it is a T cell product is based on a study showing that T cell hybridomas from minimal change disease patients release factors which can induce nephrotic syndrome in rats. A recent clue to the etiology was the surprising discovery that the podocyte can acquire characteristics of antigen-presenting cells such as dendritic cells. In particular, minimal change disease is associated with expression of the CD80 antigen in podocytes and with shedding of CD80 into the urine. To date no diagnostic tests are available, but it appears that the CD80 is tightly associated with the development of proteinuria, and urinary CD80 excretion resolves when the child undergoes remission.

172
Q

Clinical Presentation of Minimal change disease

A

The most common presenting symptom is edema. Renal function is normal and hypertension is not observed. Many patients also give a history of allergic disease. Laboratory testing reveals normal complement levels and severe hypoalbuminemia. Hematuria is uncommon. Some cases occur in association with Hodgkin’s disease, or with use of nonsteroidal agents (idiosyncratic reaction).

173
Q

Treatment of Minimal change disease

A

Patients respond rapidly to steroid therapy. Children are typically started at 2 mg/kg/d of prednisone, and often remit after several weeks. However, relapses are common. For those with multiple relapses, a course of prednisone and cyclophosphamide (12 weeks) is often curative.

174
Q

Prognosis of Minimal change disease

A

Long term prognosis is good.

175
Q

Definition of focal segmental glomerulosclerosis (FSGS)

A

FSGS is a type of nephrotic syndrome and the most common one in young adults and African Americans. It receives its name because it is characterized on light microscopy as having segmental scarring in some glomeruli. However, the defect that causes the proteinuria is diffuse and involves the entire capillary wall.

176
Q

Epidemiology of focal segmental glomerulosclerosis (FSGS)

A

This is the most common Nephrotic syndrome in Blacks and also in young adults (20- 40 yrs old). It can also develop out of a minimal change disease, typically in children who keep relapsing and become steroid resistant.

177
Q

Pathology of focal segmental glomerulosclerosis (FSGS)

A

Light microscopy shows segmental scarring (sclerosis) in some (focal) glomeruli. Immunofluourescence is usually negative but may show some nonspecific staining of IgM and C3 in the sclerotic areas. EM shows diffuse foot process fusion consistent with a generalized capillary wall defect. Early in the course the segmental scarring may not be very evident, and the disease may appear histologicially like Minimal Change Disease. However, unlike minimal change, FSGS is typically more resistant to steroids.

178
Q

Etiology of focal segmental glomerulosclerosis (FSGS)

A

Most data suggest that FSGS is also caused by a circulating factor that affects the podocyte. Recent studies suggest it might be suPAR (soluble urokinase-type plasminogen activator receptor), but more studies are needed to confirm these observations.

179
Q

Associations with focal segmental glomerulosclerosis (FSGS)

A

FSGS may be idiopathic; may be associated with a prior minimal change disease; or may be associated with heroin use (heroin nephropathy) or with HIV infection (HIV nephropathy). Rarely FSGS may also be associated with sickle cell disease, with parvo virus infection, and with obesity.

180
Q

Clinical Presentation of focal segmental glomerulosclerosis (FSGS)

A

Patients with focal glomerular sclerosis present with idiopathic nephrotic syndrome but unlike minimal change disease may be hypertensive or have microhematuria.

181
Q

Treatment of focal segmental glomerulosclerosis (FSGS)

A

Prolonged steroid therapy (6 month) results in partial or complete remission in one half of patients. Some also respond to cyclosporine therapy. Relapse is common. HIV patients respond to antiretroviral therapy and ACE inhibitors.

182
Q

Prognosis of focal segmental glomerulosclerosis (FSGS)

A

Progression to renal failure is common. Relapses may occur following transplantation.

183
Q

Definition of membranous nephropathy

A

Membranous nephropathy is an immune mediated glomerular disease associated with immune complex deposits in the subepithelial space (between the podocyte and GBM). It is called membranous because the GBM appears thickened by light microscopy.

184
Q

Epidemiology of membranous nephropathy

A

Membranous nephropathy is the most common cause of idiopathic nephrotic syndrome in older adults with peak incidence is in the fifth and sixth decades of life and it shows a male predominance.

185
Q

Pathology of membranous nephropathy

A

Renal pathology reveals varying degrees of thickening of the GBM by light microscopy. Cellularity is normal. Special sliver stains reveal the appearance of spikes along the basement membrane. These represent extensions of new basement membrane material between the immune complex deposits present on the subepithelial side of the basement membrane. Immunofluorescence microscopy reveals granular deposits of immunoglobulin and C3 and C5b-9 along the GBM in a pattern typical of immune complex disease. Electron microscopy shows electron dense subepithelial deposits, which represent the immune complexes.

186
Q

Etiology of membranous nephropathy

A

The disease is autoimmune and is mediated by an antibody directed against an antigen on the podocyte. Recently over 70 percent of idiopathic MN has been shown to be due to antibodies to the PLA2 (phospholipase A2) receptor on the podocyte. Following antibody binding, the immune complex activates complement which injures the podocyte. The immune complex is then “capped and shed” into the subepithelial space where it forms the subepithelial immune deposits characteristic of the disease.

187
Q

Associations of membranous nephropathy

A

Most (2/3) cases are idiopathic. Some cases are associated with chronic infection (especially hepatitis B), drugs (gold and penicillamine), lupus (type V membranous), and cancer (lung, breast and GI tract). Membranous nephropathy may also occur de novo following transplantation.

188
Q

Clinical Presentation of membranous nephropathy

A

MN presents most commonly as nephrotic syndrome and edema although a significant number of patients will present early in their course with non-nephrotic proteinuria. Hypertension and renal failure are uncommon at the time of presentation, but not uncommonly develop over time. Laboratory findings are those of the nephrotic syndrome. Complement levels are normal.

189
Q

Treatment of membranous nephropathy

A

Steroids and cytotoxic drugs (chlorambucil, cyclophosphamide) are commonly administered to high risk cases, such as those with severe proteinuria (>5-6 g/d) or evidence of impaired renal function. Children and women generally have a better prognosis and are sometimes followed conservatively unless they show signs of progression. ACE inhibitors are commonly used to lower proteinuria.

190
Q

Prognosis of membranous nephropathy

A

50% end stage renal failure, 25% continued nephrotic syndrome, 25% spontaneous remission over 10-20 years

191
Q

Definitions of membranoproliferative glomerulonephritis

A

MPGN refers to a histologic pattern in which there is both proliferation (usually mesangial and in a lobular pattern) as well as thickening of the GBM. It is often separated into two major types. MPGN type I is associated with immune complex deposits. MPGN type 2 is associated with complement activation in the capillary wall in the absence of immune deposits.

192
Q

Epidemiology of membranoproliferative glomerulonephritis

A

MPGN type I is most often idiopathic or associated with HCV infection. The idiopathic form affects primarily older children and adolescents and is approximately twice as common in females as in males. MPGN type II primarily is observed in adolescents and usually result from either acquired or inherited defects in the alternative complement pathway. Some cases of type II MPGN are associated with partial lipodystrophy of the face and upper body.

193
Q

Pathology of membranoproliferative glomerulonephritis

A

Light microscopy of type I shows thickening of the basement membrane, mesangial cell proliferation and a lobulated appearance of the glomerulus. Deposits of C3 are prominent on the capillary walls and in the mesangium, and IgG deposits are also present. In Type I MPGN electron microscopy shows subendothelial and mesangial deposits representing immune complexes. Type II MPGN has similar features by light microscopy, but IF only shows C3 deposits and no IgG. By EM the basement membrane is replaced in large areas by dark “dense deposits” of unknown etiology.

194
Q

Pathogenesis of membranoproliferative glomerulonephritis

A

MPGN type I is thought to be due to passive trapping of circulating immune complexes in the subendothelial and mesangial areas, followed by complement activation and leukocyte recruitment. Type II MPGN is thought to be due to a circulating nephritic factor that activates the alternative pathway of complement. It remains unclear how this activation leads to renal injury.

195
Q

Clinical Presentation of membranoproliferative glomerulonephritis

A

Whereas most patients present with nephrotic syndrome, a substantial number, approximately 20% in Type I (and 30-50% in Type II), present with a picture of acute nephritis. Hypertension is a frequent feature early in the disease even before significant loss of renal function has occurred. A substantial number of adult patients with type I MPGN have evidence of chronic hepatitis C infection with HCV and RNA present in the circulation. Many of these patients also have cryoglobulins, rheumatoid factor and low complement levels.

196
Q

Type I MPGN

A

is associated with IC deposition and activation of the classical complement pathway, with low C4 and C3 levels.

197
Q

Type II MPGN

A

is associated with a circulating IgG that can spontaneously activate complement (C3 convertase); this factor is called Nephritic Factor or C3Nef. It is associated with normal C4 levels and low C3 levels, consistent with activation of the alternative pathway of complement.

198
Q

Treatment and Prognosis of membranoproliferative glomerulonephritis

A

The prognosis of MPGN is poor with many patients going on to develop end-stage renal disease. Some studies have shown a beneficial effect of treatment with alternate day steroids in idiopathic MPGN type I and type II, particularly in children, although this may greatly exacerbate hypertension and cause significant morbidity. Anti-platelet agents are frequently used to treat adults with MPGN but recent studies question their efficacy. Patients with HCV associated disease may respond to anti-viral therapy with alpha Interferon with or without ribavirin.

199
Q

Nephrotic syndrome in secondary renal disease

A

Diabetes, lupus and amyloid are the primary systemic diseases that cause nephrotic syndrome.

200
Q

Diabetic Nephropathy and nephrotic syndrome

A

This is the most common cause of nephrotic syndrome in adults. It often begins 7-15 years after onset of type I diabetes and occasionally earlier in subjects with type 2 diabetes. Type 1 diabetics almost always have evidence for retinopathy or neuropathy at the time of presentation of renal disease. Subjects usually present with nephrotic proteinuria. Microhemature is common. Renal biopsy shows a nodular glomerulosclerosis with thickening of the basement membranes. Treatment should be focused on improving glucose control, BP control and use of ACE inhibitors that may slow renal progression.

201
Q

Systemic Lupus erythematosus and nephrotic syndrome

A

SLE can present with a variety of types of glomerular disease, with most types being a nephritic presentation. However, lupus can present with a membranous histologic pattern in which renal function is often normal or only slightly depressed and proteinuria and nephrotic syndrome are the primary manifestations. These patients may have lower titers of ANA antibodies than usual for SLE subjects. Most are treated with prednisone and mycophenolate.

202
Q

Amyloidosis and other plasma cell dyscrasias and nephrotic syndrome

A

Nephrotic syndrome in the older subject may represent a manifestation of multiple myeloma or other plasma cell dyscrasia. Subjects may present primarily with a renal presentation (nephrotic syndrome) or with systemic symptoms, such as hepatosplenomegaly, cardiac involvement with congestive heart failure symptoms, and orthostatic hypotension. Workup includes performing free plasma light chains, and serum and urine protein electrorphoresis to look for monoclonal light chains. Renal biopsy shows amyloid deposits (in amyloidosis) or nodular glomerulosclerosis (in light chain disease). Some cases of amyloidosis may not involve light chains but rather serum amyloid protein (secondary amyloidosis). Treatment is aimed at the underlying cause.

203
Q

Glomerular disease

A

should be considered in patients with proteinuria and/or hematuria. The approach to the patient with possible glomerular disease should begin with an assessment of the protein excretion in the urine and a microscopic analysis of the urine for dysmorphic red blood cells and/or red blood cell casts. When hematuria and/or proteinuria have been identified and glomerular disease is determined to be the most likely etiology, further clinical information and serologic testing can assist in the classification of the renal disorder before invasive testing (e.g. a biopsy).

204
Q

The Nephritic Syndrome

A

Although patients with glomerulonephritis can present with mild proteinuria and hematuria, classically they present with a “nephritic” syndrome. Patients with the nephritic syndrome have hematuria, dysmorphic red blood cells and/or red blood cell casts, and proteinuria. The proteinuria can range from 200 mg per day to heavy proteinuria (greater than 10 grams per day). Clinically, it is accompanied by hypertension and edema. Renal insufficiency is common and typically progressive. The primary characteristics are of the nephritic syndrome are: Reduction in GFR (an elevated serum creatinine), Active urine sediment (RBC’s, WBC’s, and RBC casts), Proteinuria (usually sub-nephrotic), Edema, and Hypertension

205
Q

Evaluation of patients with the nephritic syndrome

A

Patients may develop glomerulonephritis as an isolated renal disease or as one manifestation of a systemic disease. Several autoimmune diseases and infections are associated with the development of glomerulonephritis. The history and physical examination should particularly focus on the assessment of rashes,
lung disease, neurologic abnormalities, evidence of viral or bacterial infections, and
musculoskeletal and hematologic abnormalities. Laboratory assessment should include a complete blood count (CBC), electrolyte panel, 24-
hour urine collection for protein and creatinine clearance, and liver function tests. Proper management of the glomerular diseases requires a tissue diagnosis to confirm the clinical findings and provide information regarding the acuity and chronicity of the disease
process.
Some forms of glomerulonephritis are self-limited. In patients with progressive disease, the decline in kidney function can occur over days to weeks (“rapidly progressive glomerulonephritis”; RPGN) or it can occur over years. Patients with Goodpasture’s syndrome and ANCA associated vasculitis can also present with pulmonary hemorrhage, referred to as pulmonary-renal syndrome. RPGN and pulmonary-renal syndrome are medical emergencies that require prompt diagnosis and treatment.

206
Q

Serologies for glomerulonephritis

A
Serum complement (C3) levels are often clinically helpful to assist in the diagnosis of a specific renal disease. Further laboratory assessment may include an antistreptolysin 
(ASO) titer, antinuclear antibody (ANA), ANCA, cryoglobulins, and/or an anti-GBM antibody.
207
Q

Pathophysiology of nephritic syndrome

A

The nephritic syndrome is caused by glomerular inflammation and manifests with an “active” urine sediment (e.g. cells and/or casts). Immune-complexes (antibody bound to a target antigen) that deposit in the mesangium or in the subendothelial space generate inflammatory mediators that have access to the circulation and can cause an influx of inflammatory cells. Glomerular endothelial injury is also caused by autoantibodies to the glomerular basement membrane (anti-GBM), and with necrotizing injury of the glomerular capillaries as occurs in the antineutrophil cytoplasmic antibody (ANCA)-mediated vasculitis.

208
Q

Proteinuria in nephritic syndrome

A

results from direct damage to the glomerular capillary wall induced by immunologic mechanisms leading to an increase in protein filtration. The degree of proteinuria is generally less than 3.0 gm/day because disruptions in the capillary wall are quite focal in nature. This is in contrast to nephrotic syndrome where proteinuria generally exceeds 3.5 gm/day and the entire glomerular capillary wall has increased permeability.

209
Q

Reduction in GFR in nephritic syndrome

A

The reduction in glomerular filtration rate results from an acute, inflammatory process within the glomerulus resulting in glomerular vasoconstriction and/or occlusion or thrombosis of some glomerular capillary loops with a consequent reduction in filtrating surface area.

210
Q

Active urine sediment in nephritic syndrome

A

the excretion of red cells, white cells and red cell casts reflects glomerular inflammation and disruption of the GBM.

211
Q

Edema in nephritic syndrome

A

results from an increase in tubular reabsorption of salt and water due to reduced, glomerular perfusion with well preserved tubular function leading to expansion of extracellular fluid volume

212
Q

Hypertension in nephritic syndrome

A

is a consequence of salt and water retention.

213
Q

Basic histologic classification of glomerulonephritis

A

Glomerular diseases are generally classified by histologic patterns based on renal biopsy in which the tissue is examined by light microscopy, immunofluorescence (to look for immune reactants) and electron microscopy (to look for location of immune deposits). Un underlying disease etiology (such as lupus or Hepatitis C infection) may present with several histologic patterns, and, conversely, not all patterns have one etiology. This has made the classification system difficult to learn. The histologic findings do correlate with the clinical presentation.

214
Q

Light microscopy findings with glomerulonephritis

A

Glomeruli are examined for the presence of cellularity (hypercellular, mesangial proliferative, normocellular, etc), for the presence of scarring (sclerosis). Lesions are termed segmental if they involve part of one glomerulus; focal is the word used if only some glomeruli are involved. Crescents are a special term when there is a proliferation of cells in Bowman’s capsule. These cells consist of macrophages and parietal epithelial cells, and they often look like a crescent of a moon on cross-section. The presence of crescents correlates with the syndrome of RPGN and carries an ominous prognosis.

215
Q

Immunofluorescence findings with glomerulonephritis

A

Special stains are performed to determine if there are immunoglobulins present (IgG, IgA, and IgM) or if complement proteins are present (typically staining for C3 and C4). The pattern of staining (capillary wall or mesangial) is also helpful.

216
Q

Electron Microscopy findings with glomerulonephritis

A

Electron microscopy (EM) is done to identify the morphology of the basement membrane, to determine if there is fusion of the podocyte foot processes (a sign of nephrotic syndrome) and to determine the presence and location (mesangial, subendothelial, or subepithelial) of any immune deposits.

217
Q

Primary forms of glomerulonephritis

A

includes Post-infectious Glomerulonephritis, IgA Nephropathy, MPGN, Anti-GBM Disease

218
Q

Post-infectious Glomerulonephritis

A

Acute post-infectious GN is a type of GN that occurs after infection. It classically occurs after a group A streptococcal infection, but it may occur with other infections such as Staphylococcus endocarditis.

219
Q

Epidemiology of Post-infectious Glomerulonephritis

A

Post-strep GN typically occurs 14 days (after throat infection) to 21 days (after skin infection) with certain M strains of Group A Streptococcus. It is most common in children, and especially in parts of the world with poor hygiene and poor access to medical care. Post infectious GN may also occur with acute or subacute endocarditis, infected vascular prostheses, abscesses and empyema, and with infected ventriculoatrial shunts.

220
Q

Etiology/Pathogenesis of Post-infectious Glomerulonephritis

A

Most studies suggest that the GN is mediated by an antibody response to certain streptococcal antigens, resulting in circulating immune complexes that lodge in the glomeruli and activate the complement system.

221
Q

Pathology of Post-infectious Glomerulonephritis

A

Light microscopy shows a proliferative and exudative glomerulonephritis with infiltration of neutrophils and monocytes. Immunofluorescence (IF) shows granular deposits of IgG and C3 in the subendothelial, mesangial and subepithelial locations. EM shows the subendothelial and mesangial deposits, as well as the classic “subepithelial humps”.

222
Q

Clinical Presentation of Post-infectious Glomerulonephritis

A

Children may present with edema and sudden weight gain. Hematuria and subnephrotic proteinuria are common, and GFR is often decreasd. Hypertension may be severe. Associated laboratory findings include elevated levels of antibodies to streptococcal antigens (ASO titer, streptozyme) and decreased levels of complement components C3 with normal levels of C4 suggesting alternate complement pathway activation.

223
Q

Diagnosis of Post-infectious Glomerulonephritis

A

The diagnosis is considered in patients presenting with nephritic syndrome with a positive streptozyme test and low C3 levels. Renal biopsy is often not performed for classic presentations, but patients with severe or persistent disease may undergo renal biopsy.

224
Q

Treatment/Prognosis of Post-infectious Glomerulonephritis

A

Since post-infectious glomerulonephritis is a self-limited disease, which subsides when the offending antigen is no longer present, and irreversible structural damage rarely occurs during this interval, patients usually get better spontaneously without specific therapy for their renal disease. About 5% of patients may suffer some irreversible renal damage with persistent proteinuria and these patients can develop progressive disease. However, 95% of patients recover normal renal function and do not appear to be at any increased risk of later renal disease. Similar but less severe forms of GN may follow infections with other bacteria (subacute bacteria! endocarditis, shunt nephritis), viral illnesses, protozoal infections, etc

225
Q

IgA Nephropathy

A

A form of mesangial proliferative glomerulonephritis in which there is a predominance of IgA immune deposits in the mesangium and rarely in subendothelial areas.

226
Q

Epidemiology of IgA Nephropathy

A

IgA nephropathy is the most common type of acute glomerulonephritis now seen worldwide and usually presents in males 15 to 35 years old.

227
Q

Etiology/Pathogenesis of IgA Nephropathy

A

The disease is thought to be mediated by the deposition of IgA immune complexes to the mesangium with activation of mesangial cells via the Fc alpha receptors, resulting in cell proliferation and matrix expansion. Several studies show that subjects with IgA nephropathy have a decreased mucosal IgA immune response to oral antigen challenge, and that the antigens may gain access to the circulation where they induce a profound bone marrow IgA response, leading to circulating immune complexes. These complexes are abnormal in that the hinge region of the IgA molecule lacks a galactose; this may impede their clearance via the sialoglycoprotein receptors for IgA in the liver. Hence high levels of circulating complexes occur, which end up depositing in the glomeruli.

228
Q

Pathology of IgA Nephropathy

A

Light microscopy shows an increase in mesangial cell number and matrix; IF shows IgA, IgG, and C3 in a mesangial pattern; EM shows mesangial immune deposits.

229
Q

Clinical Presentation of IgA Nephropathy

A

Most patients present with asymptomatic microhematuria , nonnephrotic proteinuria, and normal or only mildly reduced renal function. Occasional patients have gross hematuria with red cell casts; typically these occur with the onset of a viral illness (as opposed to 2 weeks later as occurs with post-strep GN). Some patients, especially children, may present with a systemic syndrome with fever, rash, GI complaints and renal disease. Biopsy of the skin shows IgA deposits; this entity is called Henoch-Schonlein purpura. Rare patients may have severe disease with crescent formation and acute loss of renal function or nephrotic syndrome.

230
Q

Treatment and Prognosis of IgA Nephropathy

A

About 25-50% of patients will progress slowly to renal failure over 10 to 15 years. Studies suggest that progression may be slowed by angiotensin converting enzyme inhibitors. Several recent studies also suggest a potential benefit with steroids with/without azathioprine.

231
Q

MPGN

A

MPGN refers to a histologic pattern in which there is both proliferation (usually mesangial and in a lobular pattern) as well as thickening of the GBM.

232
Q

Epidemiology of MPGN

A

MPGN type I is most often idiopathic or associated with HCV infection. The idiopathic form affects primarily older children and adolescents and is approximately twice as common in females as in males.

233
Q

Pathology of MPGN

A

Light microscopy shows thickening of the basement membrane, mesangial cell proliferation and a lobulated appearance of the glomerulus. Deposits of C3 are prominent on the capillary walls and in the mesangium, and IgG deposits are also present. Electron microscopy shows subendothelial and mesangial deposits representing immune complexes.

234
Q

Pathogenesis of MPGN

A

MPGN is thought to be due to passive trapping of circulating immune complexes in the subendothelial and mesangial areas, followed by complement activation and leukocyte recruitment.

235
Q

Clinical Presentation of MPGN

A

Patients with MPGN can present with acute glomerulonephritis or nephritic syndrome. Hypertension is a frequent feature early in the disease even before significant loss of renal function has occurred. A substantial number of adult patients with MPGN have evidence of chronic hepatitis C infection with HCV and RNA present in the circulation. Many of these patients also have cryoglobulins, rheumatoid factor and low complement levels. MPGN is associated with immune complex deposition and activation of the classical complement pathway of complement, with low C4 and C3 levels.

236
Q

Treatment and Prognosis of MPGN

A

The prognosis of MPGN is poor with many patients going on to develop end-stage renal disease. Some studies have shown a beneficial effect of treatment with alternate day steroids in idiopathic MPGN, particularly in children, although this may greatly exacerbate hypertension and cause significant morbidity. Patients with HCV associated disease may respond to anti-viral therapy with alpha Interferon with or without ribavirin.

237
Q

Anti-GBM Disease

A

Anti-GBM nephritis can cause a severe, rapidly progressive GN, which may be accompanied by pulmonary hemorrhage (Goodpasture’s syndrome) or may be renal-limited. Goodpasture’s syndrome consists of the triad of pulmonary hemorrhage, iron deficiency anemia and GN associated with circulating antibody to GBM. It is a rare disease of young males.

238
Q

Pathology of Anti-GBM Disease

A

Glomerulonephritis results from binding of antibody to an antigens in type IV collagen within the GBM to produce linear IgG deposits by IF, complement activation, neutrophil infiltration and the other clinical and morphologic consequences of acute immune glomerular injury. The disease is often associated with extensive crescent formation and rapid loss of renal function (RPGN). Clinical Presentation. Pulmonary hemorrhage usually precedes renal involvement and is now thought to be a consequence of antibody deposition on alveolar basement membrane, which occurs when some form of lung injury is present at the time anti-GBM antibody develops, in the circulation. Some patients have anti-GBM nephritis without pulmonary hemorrhage. Smoking and solvent exposure have both been implicated in causing lung damage that allows antibody deposition. Iron deficiency anemia is a consequence of extensive intrapulmonary hemorrhage. Anti-GBM antibody can be detected in the circulation using an ELISA assay. There are no other serologic abnormalities in anti-GBM nephritis.

239
Q

Treatment and Prognosis of Anti-GBM Disease

A

Early diagnosis of this form of GN is important since there is good evidence that aggressive treatment with steroids, immunosuppressive agents and plasma exchange substantially increases renal survival if therapy is initiated before oliguria (urine output of less than 500 cc/day) develops or the serum creatinine exceeds 6 mg/dl. A kidney biopsy is usually performed even when anti-GBM antibody levels are elevated to establish the severity and reversibility of the disease and to assess its likelihood of response to aggressive treatment. If a kidney transplant is carried out in the presence of circulating anti-GBM antibody, a recurrence of the disease in the transplanted kidney is common

240
Q

Glomerulonephritis and Systemic Diseases

A

Pauci-Immune Renal Vasculitis, Lupus Nephritis, Cryoglobulinemia, Cryoglobulinemia, and Henoch-Schonlein purpura (HSP)

241
Q

Pauci-Immune Renal Vasculitis

A

Small vessel vasculitis frequently involves the kidneys. Several diseases can cause immune-complex mediated renal vasculitis (e.g. cryoglobulinemia, lupus, and anti-GBM disease). Patients with small vessel vasculitis of the kidneys who do not have evidence of immune-complex deposition in vessels are considered to have pauci-immune vasculitis.

242
Q

Pathology of Pauci-Immune Renal Vasculitis

A

Histologically, the glomeruli in patients with renal involvement in all forms of pauci- immune small vessel vasculitis demonstrate fibrinoid necrosis and crescents. Immune complexes must be absent in order to make the diagnosis of pauci-immune vasculitis.

243
Q

Pathogenesis of Pauci-Immune Renal Vasculitis

A

Approximately 90% of patients with pauci-immune small vessel vasculitis have detectable anti-neutrophil cytoplasmic antibodies (ANCA). ANCAs recognize several different antigens, including myeloperoxidase (MPO) and proteinase-3 (PR-3). There is evidence that ANCA are pathogenic in small vessel vasculitis.

244
Q

Clinical Presentation of Pauci-Immune Renal Vasculitis

A

All forms of pauci-immune small vessel vasculitis can affect multiple organ systems, including the skin, lungs, and gastrointestinal system. The clinical presentation of small vessel pauci-imune vasculitis is variable, but patients generally present with a nephritic pattern of renal disease. The renal disease can progress rapidly, making it very important to diagnose the disease promptly. Because the lungs are frequently involved in all forms of ANCA associated vasculitis, patients can present with alveolar capillaritis and pulmonary hemorrhage (“pulmonary-renal syndrome”).

245
Q

Treatment and Prognosis of Pauci-Immune Renal Vasculitis

A

Whether systemic or primary, patients with pauci-immune small vessel vasculitis are treated with immunosuppressive drugs. The most commonly used protocols include high-dose steroids and cyclophosphamide (either oral or intravenous). Plasma exchange may be beneficial in patients with pulmonary hemorrhage and in patients with renal failure severe enough to require dialysis. Recent studies have demonstrated that rituximab is as effective as cyclophosphamide for inducing remission in patients with severe disease.

246
Q

Epidemiology of Lupus Nephritis

A

More than half of the patients with lupus develop clinically evident renal involvement. Renal disease is an important cause of morbidity in these patients, and mortality is higher in patients with lupus who have renal involvement than in those who do not.

247
Q

Pathology of Lupus Nephritis

A

Immune-complexes may be seen within the mesangium, the subendothelial space, and the subepithelial space. Immunofluorescence may demonstrate C3, IgG, IgM, IgA, and C1q all within the same kidney. These deposits appear as “lumps and bumps” and are distinguishable from the linear pattern seen in anti-GBM disease.

248
Q

Pathogenesis of Lupus Nephritis

A

Lupus is caused by the loss of tolerance to self-antigens and the generation of autoantibodies. Most of the autoantibodies react with antigens present in the cell nucleus, such as DNA, RNA, and histone. Pre-formed immune complexes may deposit in the kidney, or the antibodies and antigen may deposit separately.


249
Q

Clinical Presentation of Lupus Nephritis

A

The manifestations of lupus nephritis are variable among patients, and in individual patients the nature of the disease can change over time and in response to therapy. Renal involvement is usually discovered by the detection of proteinuria and hematuria, but patients can present with either nephritic or nephrotic patterns of injury. Because the histologic pattern of injury in lupus nephritis is variable, different classifications have been developed in order to better predict the prognosis.

250
Q

Treatment and Prognosis of Lupus Nephritis

A

in general, therapy for lupus nephritis includes high-dose corticosteroids in combination with either mycophenolate mofetil or cyclophosphamide, particularly for the treatment of diffuse proliferative lupus nephritis.

251
Q

Cryoglobulinemia

A

Cryoglobulins are antibodies that precipitate in the cold. In vivo they can form immune-complexes that precipitate in small vessels, causing vasculitis.

252
Q

Epidemiology of Cryoglobulinemia

A

Cryoglobulinemic glomerulonephritis should be suspected in any patient with known hepatitis C infection who develops renal disease.

253
Q

Pathology of Cryoglobulinemia

A

In the kidney, cryoglobulinemia causes an immune-complex glomerulonephritis. On renal biopsy, affected patients usually have a membranoproliferative pattern of injury and subendothelial immune deposits. Microtubular structures are seen on electron microscopy, and the deposits can form a characteristic “fingerprint” appearance.

254
Q

Pathogenesis of Cryoglobulinemia

A

Cryoglobulins are most frequently associated with hepatitis C infection, although they are also seen in other conditions including lymphoproliferative disorders, autoimmune disease (particularly Sjögren’s syndrome), and other infections. Cooling of blood in the extremities may favor precipitation of cryoglobulins in blood vessels. In organs such as the kidneys, immune complexes formed by binding of IgM with rheumatoid activity may favor precipitation.

255
Q

Clinical Presentation of Cryoglobulinemia

A

Cryoglobulinemia can affect numerous different tissues throughout the body. Most patients with symptomatic disease develop palpable purpura, arthralgias, and generalized weakness. Patients typically have proteinuria, hematuria, and slowly progressive disease. Some patients have nephrotic range proteinuria, however, and patients can have a rapid loss of renal function. Labs that support the diagnosis of cryoglobulinemia include a low C4 level and the cryoglobulins often have rheumatoid factor activity.

256
Q

Treatment and Prognosis of Cryoglobulinemia

A

For patients with hepatitis C and symptomatic cryoglobulinemia, antiviral therapy with peginterferon alpha and ribavirin is associated with clinical improvement. B cell depleting therapies, such as rituximab, are beneficial in patients with an underlying B cell lymphoproliferative disease and in those with rapidly progressive or resistant disease. Plasmapheresis removes the cryoglobulins and can be beneficial in patients with rapidly progressive disease.

257
Q

Henoch-Schonlein purpura (HSP)

A

The classic clinical manifestations of HSP in children include skin lesions (palpable purpura), arthritis, GI involvement (colic and bleeding) and glomerulonephritis. In adults renal involvement is often more severe and systemic disease less obvious. The renal lesion is similar to IgA nephropathy but more severe with a focal proliferative necrotizing glomerulonephritis, often with crescent formation, accompanied by mesangial and capillary wall deposits of IgA. IgA nephropathy and HSP are believed to be caused by similar mechanisms involving IgA immune complexes.

258
Q

Clinical Syndromes of Glomerular Disease

A

Asymptomatic hematuria/ proteinuria, Acute nephritic syndrome (hematuria/proteinuria + ARF), Rapidly progressive nephritic syndrome (RPGN), Nephrotic syndrome with Massive proteinuria (>3.5 g/24hrs) not compensated by hepatic albumin synthesis) and Decreased oncotic pressure – edema, and Chronic renal failure

259
Q

Acute Nephritic Syndrome

A

increased glomerular capillary permeability leading to hematuria and proteinuria. Decreased GFR leading to Na+ and H2O retention (brought on by edema, CHF, hypertension), azotemia, and hyperkalemia

260
Q

Rapidly Progressive Nephritic Syndrome

A

increased glomerular capillary permeability causing hematuria and proteinuria. GFR is greatly decreased leading to more fluid retention, more azotemia, and oliguria

261
Q

Nephrotic Syndrome

A

Massive proteinuria (>3.5 gms/24hrs) that is not compensated by hepatic albumin synthesis. Decreased plasma oncotic pressure leading to Na+ and H2O retention, which causes massive edema, hypercholesterolemia, etc.

262
Q

Morphologic Patterns of Glomerular Disease

A

Cell proliferation occurs in possibly the mesangial, endocapillary, and epithelia (podocyte) crescents. Leukocytic infiltration. Basement membrane thickening and changes. Segmental or global sclerosis

263
Q

Characterization of immunopathogenesis of glomerular disease

A

Immunofluorescence can detect Ig’s/ complement, show distribution (mesangial, capillary), and illuminate pattern (linear, granular). Electron microscopy can show Deposits of Ag – Ab complexes and GBM changes

264
Q

Serum characterization of glomerular disease

A

Serum can show Complement levels (decreased in some acute GN’s), ANA (antinuclear antibodies-SLE), ASO (anti-streptolysin 0), or ANCA (anti-neutrophil cytoplasmic Abs – Wegener’s, PAN)

265
Q

Types of nephritic disease

A

Benign familial hematuria (thin BM dis), Alport’s Disease, IgA nephropathy, Postinfectious GN, Focal necrotizing/crescentic GN, and Lupus GN

266
Q

Thin Basement Membrane Disease

A

Also known as benign familial hemauria (excellent prognosis). Occurs in 1% of the population. Mutations in genes encoding collagen IV. Need to differentiate from Alport’s syndrome. Diagnosis based on electron microscopy

267
Q

Pathogenesis of Glomerular Injury- Non-inflammatory mechanisms

A

Circulating factors/Ig’s bind to GEC membranes &/or GBM without fixing complement. Loss of polyanion (charge selective). GEC (GEC=glomerular epithelial cell) detachment from GBM (size selective). Eg: minimal change dis/focal sclerosis. Complement-fixing anti-GEC Ab’s. Alternative pathway. C5b-9 leading to increase perm of GBM (size selective). May involve oxidants/proteases from GEC. Eg: membranous nephropathy

268
Q

Alport syndrome

A

a nephritic syndrome caused by a defect in type IV collagen. Most cases are X-linked, preferentially affecting males. That classic clinical triad of Alport’s syndrome is: Hematuria, sensorineural hearing loss ocular abnormalities (e.g., lens dislocation, posterior cataracts, corneal dystrophy). Urinalysis findings that are found in Alport’s syndrome include: RBC casts, Hematuria, proteinuria, and pyuria. A renal biopsy in Alport syndrome will show a classic “basketweave” appearance on electron microscopy.This is due to the irregular thickening/thinning of the glomerular basement membrane with splitting/lamination of the lamina densa. The mainstay treatment for Alport syndrome includes ACEIs and ARBs.

269
Q

Proliferative Glomerulonephritis

A

Hypercellularity in Endocapillary (occlusion of capillary loops), Mesangial, and Epithelial – crescents (reaction to severe injury to glomerular capillaries). Inflammatory cells

270
Q

IgA nephropathy

A

the most common cause of glomerular hematuria world-wide.The exact cause is unknown, but it may be related to infection. IgA immune complexes deposit in the mesangial cells of the kidneys, leading to damage of the glomeruli. IgA nephropathy tends to occur within one week of URI, whereas PSGN typically occurs severalweeks post-infection. IgA nephropathy presents with normal complement levels, whereas PSGN presents with hypocomplementemia (particularly, C3 complement). Throat culturefor Group A Streptococcus willbe positive in PSGN following a URI. This is typicallynot the casein IgA nephropathy. Increased proliferation of mesangial cells and immune complexes are seen on electron microscopy. Light microscopy may show normal-appearing glomeruli or mesangial widening due to mesangial proliferation and/or deposition of immune complexes Immunofluorescence microscopy. The urine dipstick will be positive for bloodand protein. The urine sediment can show red blood cells and red blood cell casts. IgA nephropathy is treated with ACEIs or ARBs as well as statins. If nephrotic range proteinuria develops, steroids are used.

271
Q

Henoch Schönlein Purpura (HSP)

A

also known as IgA vasculitis, is the most common cause of systemic vasculitis in children. HSP is a small vessel vasculitis (arterioles, capillaries, and venules) characterized by IgA, C3, and fibrin deposition in blood vessels. HSP and IgA nephropathy are considered to be related disorders. In about 50% of cases, the patient (usually a child between ages 2 and 11) will have a recent history of a upper respiratory infection (often viral or Group A Streptococcus). HSP is associated with the following tetrad of clinical manifestations: Palpable purpura in patients that have neither thrombocytopenia or coagulopathy, Polyarticular arthritis, Abdominal pain, and Renal failure. Biopsy of the purpuric skin rash will demonstrate IgA deposition in the vasculature, similar to the findings associated with a renal biopsy and is generally considered adequate for making the diagnosis. A kidney biopsy will show mesangial deposition of IgA, but is generally reserved for patients with severe renal involvement. It will show IgA in the mesangium, similar to IgA nephropathy. Urinalysis will be positive for blood and protein in the urine. The microscopic sediment may demonstrate red cells and red cell casts.

272
Q

PSGN (poststreptococcal glomerulonephritis)

A

Upper respiratory infection or skin infection with a nephritogenic strain of group A β-hemolytic Streptococcus pyogenes (especially serotype M12, M4, or M1). PSGN presents 2-3 weeks after the infection due to the time togenerate antibodies and form immune complexes.

Note that the delayed onset ofglomerulonephritisin PSGN differentiates it from IgA nephropathy, which presents with”synpharyngitic glomerulonephritis” (concurrentpharyngitis and glomerulonephritis). Subepithelial (subpodocyte) immune complex deposition (type III hypersensitivity) in glomeruli with subsequent activation of the alternative complement pathway: Decreased serum C3 level; normal C1 and C4 levels and C5a recruits neutrophils → glomerular damage/inflammation (i.e., glomerulonephritis) with large hypercellular glomeruli due to WBC infiltrate and proliferation of mesangial and endothelial cells. Note: renal tubules are not the primary target during acute attacks of glomerulonephritis, therefore renal tubular function (concentrating ability) remains intact.

273
Q

Microscopic appearance of renal biopsy of PSGN

A

Light microscopy:

  • Large hypercellular glomeruli. Immunofluorescence microscopy:
  • Granular/coarse deposits of IgG, IgM, and C3 along the GBM (glomerular basement membrane) and in the mesangium. “starry sky”. Electron microscopy:
  • Large subepithelial (subpodocyte) electron-dense deposits—prominent “bumps and humps”
  • Subendothelial, intramembranous, and mesangial electron-dense deposits may also be visualized but are less common and less prominent than the subepithelial deposits
274
Q

Crescentic Glomerulonephritis

A

Crescents: histologic sign of severe acute glomerular disease. Caused by fibrinoid necrosis of capillaries. Clinically present as RPGN. % of glomeruli with crescents correlates with serum creatinine levels and prognosis. Glomeruli will usually heal with a scar. Diverse etiologies

275
Q

Diseases with linear staining, Focal Segmental Necrotizing and Crescentic Glomerulonephritis

A

Goodpasture’s Syndrome and Anti-GBM

276
Q

Diseases with granular staining, Focal Segmental Necrotizing and Crescentic Glomerulonephritis

A

IgA, SLE, Endocarditis, and Idiopathic

277
Q

Diseases with no staining, Focal Segmental Necrotizing and Crescentic Glomerulonephritis

A

Wegener’s, Microscopic PAN, Churg-Strauss, and Idiopathic

278
Q

Focal Segmental Necrotizing and Crescentic Glomerulonephritis Serum/ blood studies

A

Goodpastures: anti-GBM Abs. Lupus: ANA, anti-dsDNA, complement depletion (C3 & C4). Vasculitis: ANCA (ex. MPO – cationic protein that can fix complement). Endocarditis: blood cultures

279
Q

Anti-GBM Disease

A

Nephrotoxic IgG antibody. Rare autoimmune disease – Antibodies directed against the α-3 chain of collagen IV in the basement membrane of tissues. If renal limited, anti-GBM disease. If renal and lung involved, Goodpasture’s Syndrome. Classic presentation is of RPGN and hemoptysis. Classic linear (ribbon-like) appearance on immunofluorescence (IF)

280
Q

Crescentic GN

A

usually is RPGN. Can have variable pathogenesis including immune complexes, anti-GBM, SLE, or idiopathic. Therapy includes steroids, cytotoxics, or plasmaphoresis. Prognosis is poor.

281
Q

Lupus nephritis

A

is thought to arise from autoimmune deposition and destruction of the glomerulus and glomerular basement membrane. More specifically, anti-dsDNA and anti-phospholipid antibodies play a major role in the development of nephritis.In addition to symptoms specific to lupus nephritis, patients may also exhibit general symptoms of SLE including rash, oral ulcers, arthritis, synovitis and serositis. In addition to history and physical, lupus nephritis is diagnosed based on labs such ashematuria, proteinuria, andpositive SLE serology (ANA, anti-DNA antibodies). Renal biopsy should be considered in any patient with symptoms of active nephritis. In general, active nephritis is associated with anelevated ESR and anti-dsDNA antibodies with depressed C3 and C4 levels. Stages I and II of lupus nephritis require no treatment, only observation. Classes III, IV and V require more aggressive treatment with steroids and immunosuppressive agents, mainly cyclophosphamide. ACE inhibitors are a standard of care to reduce proteinuria.

282
Q

Class I LN

A

No or minimal changes by light microscopy. Asymptomatic.

283
Q

Class II LN

A

Mesangial glomerulitis. Minimal renal disease. A. Normal by light microscopy. Mesangial deposits by EM and/or IF. B. Mild to moderate mesangial hypercellularity/sclerosis

284
Q

Class III LN

A

Focal, segmental glomerulonephritis. (Mesangial GN with up to 55% of glomeruli with segmental lesions). Increased disease, may have hypertension.

285
Q

Class IV LN

A

Diffuse glomerulonephritis. (Over 55% of glomeruli with seg/global lesions, MPGN). Increased disease, may have hypertension.

286
Q

Class V LN

A

Membranous nephropathy. Heavy proteinuria.

287
Q

Class VI LN

A

or advanced fibrosis, the kidney disease is irreversible and is likely not amenable to immunosuppressive treatment.

288
Q

Minimal-Change Disease

A

(Lipoid Nephrosis; Nil disease) is the mostcommon cause nephrotic syndrome in children, but it can still occur in adults. Most cases are idiopathic. The classic clinical presentation of minimal change disease is a young child (most commonly 2-6 years of age), sometimes with a recent history of respiratory infection or routine prophylactic immunization who presents with massive proteinuria and S/Sx of nephrotic syndrome. Hodgkin’s disease and non-Hodgkin’s lymphoma have been associated with minimal change disease. A kidney biopsy is required to definitively diagnose minimal change disease. Light microscopy findings will be normal. Electron microscopy will show: Diffuse effacement of podocyte (visceral epithelial cell) foot processes. Normal-appearing glomerular basement membrane, no electron-dense deposits. Lab findings of minimal change disease are similar to those of other nephrotic syndromes: Hyperlipidemia, Hypoalbuminemia, and Heavy proteinuria. Minimal change disease normally responds very well to corticosteroids.

289
Q

The pathogenesis of FSGS

A

involves damage (possibly cytokine-mediated) and focal disruption of podocytes (visceral epithelial cells). This epithelial damage results in foci of increased permeability, thereby allowing entrapment of plasma proteins, ultimately causing the sclerosis and hyalinosis observed on light microscopy.

290
Q

Microscopic findings of Focal segmental glomerulosclerosis (FSGS)

A

Light microscopy shows segmental sclerosis (but no deposits) andhyalinosis withinsome glomeruli. Some glomeruli are unaffected and thus appear normal. Immunofluorescence microscopy (IF) is negative, as no immune complex deposits are visualized. Despite a negative IF, IgM and C3 may be seen in sclerotic areas and/or in the mesangium. Electron microscopy shows diffuse effacement of foot processes in both sclerotic and non-sclerotic areas.

291
Q

Risk factors associated with FSGS

A

can be remembered with the mnemonicMOSAIC*: Minority (African American or Hispanic). Obesity. Sickle cell disease. AIDS (HIV). IV drug abuse (heroin) andInterferon treatment. Chronic kidney disease(secondary to congenital absence or surgical removal). *FSGS looks like a “mosaic” on histology since it shows a focal/segmental pattern of sclerosis.

292
Q

Membranous nephropathy

A

involves the sub-epithelialdeposition of immune complexes, which cause activation of complement cascade and subsequent release of proteases and oxidants. This contributes to damage to the glomerular capillary wall, leading to proteinuria.

293
Q

Membranous nephropathy (membranous glomerulopathy)

A

is the most common nephrotic syndrome in Caucasian adults.

294
Q

Membranous nephropathy pathophysiology

A

On light microscopy theglomeruli appear normal in the early stages of the disease, whilelate in the disease a thickened glomerular basement membrane and glomerular capillary wall may be present. Immunofluorescence microscopy will revealgranular deposits of IgG and/or C3. Electron microscopyshows subepithelial (subpodocyte) immune complex deposits that form aspike and dome pattern. Thespikes are basement membrane material and the domes are immune complex deposits. The majority of patients with primary membranous nephropathy have circulating autoantibodies to the renalphospholipase A2receptor. Although 85% of cases of membranous nephropathy are primary idiopathic, there are a variety of causes of secondary membranous nephropathy: Loss of antithrombin III in membranous nephropathycauses a hypercoagulable state that may lead torenal vein thrombosis (RVT). Of all causes of nephrotic syndrome,RVT is most commonly associated withmembranous nephropathy.

295
Q

Membranoproliferative glomerulonephritis (MPGN)

A

involves thedeposition of immune complexesin the glomerular basement membrane (GBM), which induces a duplication (commonly referred to as “splitting”)of the GBM, resulting in the classical “tram-track” appearance. Patients with primary/idiopathic MPGN most often present in childhood or young adulthood. MPGN type I involvessubendothelial immune complex deposits of complement components (C1, C4, IgG) that lead to activation of both classical and alternative complement. This results in decreased serum levels of C1, C4, and C3. Causes of MPGN type I include: Autoimmune diseases: SLE (systemic lupus erythematosus) and SS (sjögren’s syndrome). Infection: Hepatitis B, Hepatitis C–usually with cryoglobulinemia, HIV, and Schistosomiasis. α1-Antitrypsin deficiency. Malignancy: CLL (chronic lymphocytic leukemia), and Lymphoma. MPGN type II involves intramembranous immune complex depositsof C3 andproperdin. These are also known as“dense deposits.”

296
Q

Pathogenesis ofMPGN type II

A

C3 nephritic factor (C3NeF), an autoantibody,binds C3 convertase, thereby preventing itsinactivation. Constitutive activity ofC3 convertase catalyzessystemicactivation of the alternative pathway of the complement cascade and thepersistent breakdown of C3. Glomerular deposition of inflammatory mediators, including C3 and IgG induces inflammation. SinceC3 nephritic factorstabilizesC3 convertase, thereby enhancing C3 activation and consumption,patients withMPGN type IIhave a decreased levels of serum C3 only(patients have normal levels of C1 and C4). In addition todecreased serum C3, patients withMPGN type IIwill havedecreased levels of Factor B and properdin, since they are components of the constitutively activealternative complement pathway.

297
Q

Chronic infections associated with MPGN

A

Infected ventriculoatrial shunts, Infective endocarditis, Chronic suppuration. Schistosomiasis, Malaria, and Hepatitis B

298
Q

Neoplasms associated with MPGN

A

Epithelial tumors, Leukemia and malignant lymphoma, Macroglobulinemia, Cryolobulinemia, and Multiple myeloma

299
Q

Systemic connective tissue disorders associated with MPGN

A

Systemic lupus, Polyarteritis, Sjogren’s syndrome, and Henoch-Schonlein purpura

300
Q

Miscellaneous conditions associated with MPGN

A

Hepatic cirrhosis,Toxic epidermal necrolysis, Alpha-antitrypsin deficiency, Sickle cell disease, Kartagener’s syndrome, Kleinfelter’s syndrome, and Heroin abuse

301
Q

Renal Amyloidosis

A

Kidney is involved in 85% of cases. AA vs AL (light chain types). Usually present with proteinuria. Histology shows amorphous fluffy pink material in glomeruli and vessels. Positive on Congo Red stain with apple green birefrigencence

302
Q

Diabetes Mellitus Kidney Lesions

A

includes hyaline arteriolar disease and diabetic glomerulosclerosis.

303
Q

Hyaline arteriolar disease

A

resembles HTN but more severe. Can occur without HTN (multifactorial).

304
Q

Diabetic glomerulosclerosis

A

Diffuse or nodular expansion of the mesangium. Mesangial “lysis. Basement membrane thickening. Consequence of altered glucose and insulin metabolism: Nonenzymatic glucosylation of proteins (mesangial matrix, GBM, etc.) and Hyperfiltration

305
Q

Other pathology associated with diabetes

A

Vascular atherosclerosis and arteriolosclerosis. Increased risk of pyelonephritis (acute and chronic): Renal scarring and Renal papillary necrosis (Blood supply to the papillae is tenuous)

306
Q

Hypertensive Renal Disease

A

Finely granular surface (scarred glomeruli). Blood vessels have Medial and intimal thickening and Hyaline deposition

307
Q

Malignant (accelerated) Hypertensive Renal Disease

A

Initial event is renal vasculature injury. Result is fibrinoid necrosis and hyperplastic arteriolitis. Kidneys respond by secreting more renin and perpetuating the problem

308
Q

Renal amyloidosis

A

Subendothelial and mesangial amyloid deposits. Amyloidosis is identified by Congo red stain. It showsapple green birefringenceunder polarized light. Primary amyloidosis of the amyloid light chain (AL) type is associated with plasma cell disorders such as multiple myeloma. Protein AA type is associated with inflammatory diseases such as rheumatoid arthritis.

309
Q

Creatinine for determining GFR

A

fairly constant with relation to muscle mass. Not bound to plasma proteins and is therefore filtered freely. It is also not reabsorbed by renal tubules. Small amount secreted by renal tubules (10%).

310
Q

Polyuria

A

greater than 2 L in 24hours of urine. Increased urine volume due to defective hormonal regulation of volume homeostasis (ADH), defective renal salt/water absorption, or osmotic diuresis (DM).

311
Q

Oliguria

A

less than 500ml in 24 hours. Decreased urine volume either due to prerenal, postrenal, or renal parenchymal disease

312
Q

Anuria

A

less than 100ml in 24 hours.

313
Q

Yellow-green-brown urine

A

bile pigments (mostly bilirubin) (dark foam if shaken, concentrated urine with white foam).

314
Q

Orange-red-brown urine

A

excreted urobilinogen

315
Q

Pink-red urine

A

hematuria, hemoglobinuria, myoglobinuria, porphyrias, beet ingestion

316
Q

Dark brown/ black urine

A

methemoglobin, rhabdomyolysis ( cola-colored), L-dopa, homogentisic acid (alkaptonuria).

317
Q

Specific gravity of urine

A

relative proportions of dissolved solids per volume (density, weight, number). Urea, NaCl, sulfate, phosphate. Reagent strip vs. refractometer vs. urinometer. Vs. falling drop method. Its helpful when the patient has acute oliguria. Helpful with subtle other abnormalities. Slight levels of protein or cells may be dismissed in the setting of a high specific gravity.

318
Q

Osmolality of urine

A

number of particles of solute per volume of solution.

319
Q

Proteinuria

A

normal is less than 0.5g/day. Types of increase include postural proteinuria, proteinuria in the elderly, glomerular disease (nephrotic syndrome is over 3.5g/day), overflow (multiple myeloma). Proteins have a net charge and the color changes depending on the concentration of protein present, due to pH changes.

320
Q

Glucose in urine

A

filtered by the glomerulus and nearly entirely reabsorbed. Positive in diabetes, pregnancy, endocrine disorders, and pancreatic disorders. Serum glucose and diabetes testing should follow positive. False positive if collection jar is left open. Vitamin C can lead to false negative results. Copper deduction test can detect reducing sugars. Thin layer chromatography for specific sugar ID.

321
Q

Fructose in urine

A

inherited enzyme deficiency

322
Q

Galactose in urine

A

inherited enzyme deficiency

323
Q

Lactose in urine

A

pregnancy/ lactation, enzyme deficiency.

324
Q

Pentose in urine

A

eating lots of fruit

325
Q

Sucrose in urine

A

diet, enzyme deficiency, factitious

326
Q

Ketones in urine

A

product of lipid metabolism, normally undetectable. Positive anytime there is an increase in lipid metabolism such as with diabetes, alcoholism, cirrhosis, prolonged fast heavy exercise. Dipsticks detect mostly acetoacetic acid, are less sensitive to acetone, don’t detect hydroxybutyrate. Acetoacetic acid and acetone react with nitroprusside to create a colored compound. Diabetic ketonuria is up to 50mg acetoacetic acid/dL urine before symptoms of ketosis. Nondiabetic ketonuria symptoms include fever, vomiting, cachexia. There can be false positives with some medications such as L-dopa, phenolphthalein.

327
Q

Blood in urine

A

dipstick tests for the peroxidase-like activity of hemoglobin, interference from vitamin C. must differentiate based on history and other tests between hemoglobinuria, myoglobinuria, or hematuria. Microscopy can help differentiate type of hemo or myo-globinuria. No RBCs (free hemoglobin or myoglobin) in the urine is usually due to intravascular hemolysis. Free RBCs (non-glomerular hematuria is usually from kidney (RCC), renal pelvis, ureter, bladder, or urethra. They can also be due to contaminant (hemorrhoids or vaginal bleeding). RBC casts (glomerular hematuria) due to renal dysfunction, nephritic syndrome

328
Q

Hemosiderin in urine

A

an iron-storage complex. Can appear 2-3 days after hemolytic episodes. Renal tubules catabolize hgb to ferritin and hemosiderin. There are yellow-brown granules with Prussian blue stain.

329
Q

Urinary bilirubin

A

is absent in normal sample. With obstruction to bile flow, it is increased causing dark urine. It is increased early on in liver damage, hepatitis, or cholestasis. It is absent in hemolysis, hemolytic anemia.

330
Q

Urobilinogen

A

it is present normally but undetectable. If a neoplasm is causing obstruction to bile flow, it is low and undetected, with gallstones, it is variable. With liver damage, hepatitis, or cholestasis, it is decreased early and increased late. With hemolysis, hemolytic anemia, it is increased.

331
Q

Fecal color in jaundice

A

normal finding is dark. With obstruction to bile flow, it is pale (intermittent with gallstones in common bile duct; persistent with neoplasm in duct or pancreas). Appears pale early and dark later in hepatitis; pale with cholestasis. It is also dark with hemolysis, hemolytic anemia

332
Q

Indirect tests for urinary tract infection

A

nitrite and leukocyte esterase

333
Q

Nitrite in urine

A

Nitrate is reduced to nitrite by many bacteria and appears in urine. Positive result strongly suggests gram negative bacteria in the urine (high specificity). Exception is if the container has been open to air for a long time. Negative result does not help (low sensitivity). Not all bacteria that cause UTI have the ability to generate nitrite (Enterococci). Urine must be present in bladder for a few hours. Vitamin C, urobilinogen, low pH are interfering substances.

334
Q

Leukocyte esterase in urine

A

neutrophils make leukocyte esterase at part of the lysosomal hydrolyzing mechanism to destroy engulfed organisms. Leukocyte esterase is an indirect measure of the number of neutrophils present in the urine sample. Higher sensitivity but lower specificity (for UTI) in comparison to nitrite. Other sources of neutrophils from a UA include vaginal secretions or glomerulonephritis. Other sources of esterase include trichomonads and eosinophils and can lead to false positive results. Chronic inflammatory cells (e.g. lymphocytes) will not be detected, chronic cystitis may be missed.

335
Q

Oval fat bodies in urine

A

renal tubule epithelial cells with lipid. They are 2-3*RBC diameter. Degenerated tubular cells containing abundant lipid, which appears refractile. With polarized light, larger granules glow as maltese crosses. Can form fatty carts. Indicates nephrotic syndrome. May also contain heme pigment of hemosiderin.

336
Q

Casts in urine

A

cylinder of tamm-horsfall protein which has congealed and taken on the form of the tubule. Will incorporate whatever is also in the tubule at the time of formation. Will usually have two smooth parallel edges and blunt ends. Matrix casts include hyaline and waxy casts. Cellular casts include red, white, and tubular cell casts. Inclusion casts include granular, fatty, and crystal casts.

337
Q

Hyaline casts

A

appear clear, colorless, very hard to see. Is nonspecific, a few are normal. Have rounded ends and parallel edges. They are increased in dehydration, physical excertion, fever, or renal injury (only if in large quantity).

338
Q

Waxy casts

A

they have a high refractive index, easier to see than hyaline casts. Has charp margins, blunt ends, and cracks in lateral margins. Associated with advanced chronic renal failure. 8-12 RBC diameters wide.

339
Q

Red cell casts

A

signify glomerular disease. Lumpy edges. Contain anucleate, slightly reddish, pale discs. Establishes kidney as source of bleeding, not lower urinary tract.

340
Q

White blood cell casts

A

signify inflammation (e.g. pyelonephritis, allergic interstitial nephritis, interstitial nephrititis). Composed of WBCs, look for lobed nuclei of neutrophils.

341
Q

Tubular cell casts

A

almost entirely renal tubular cells, look for singular round nuclei. Distinguish from white cell casts. Suggest acute tubular necrosis, viral disease, or drug exposure.

342
Q

Granular casts

A

trapped cellular debris or protein aggregates. Made up of immune complexes and fibrinogen.

343
Q

Fatty casts

A

from oval fat bodies. From nephrotic syndrome

344
Q

Crystal casts

A

urates, calcium, oxalates, sulfanaimdes

345
Q

Yellow red casts

A

hemoglobin

346
Q

Yellow brown casts

A

Hemosiderin, or bilirubin

347
Q

Red brown casts

A

myoglobin

348
Q

Acidic crystals

A

includes amorphous urates (pink-red-brown granules) and uric acid crystals

349
Q

Uric acid crystals

A

variety of shapes including rhombic or four sided flat plates, prisms, oval forms. Most are colored, usually yellow or red-brown. Small numbers common and nonspecific. Large bumbers indicate increased nucleoprotein turnover, especially with chemotherapy. Also seen with lesch-nyhan (overproduce uric acid—hypoxanthine guanine phosphoribosyl-transferase deficiency).

350
Q

Calcium oxalate

A

neutral crystals. Small colorless octahedron. Resemble envelopes. Most common component of renal stones. Excreted in ethylene glycol toxicity.

351
Q

Alkaline cystals

A

amorphous phosphates (white granules) and triple phosphate

352
Q

Triple phosphate

A

ammonium magnesium phosphate, struvite. Looks like coffin lids. Form majority of staghorm calculi. Related to proteus mirabilis urinary tract infection.

353
Q

Cysteine crystals

A

flat hexagonal plates, don’t polarize. Rare but clinically important. Occur in patients with cystinuria, may be associated with severe and recurrent cysteine calculi, autosomal recessive.

354
Q

Other abnormal crystals

A

tyrosine crystals (black clumps or sheaves; seen in severe liver disease), leucine crystals, sulfonamide (sulfadiazine) crystals, ampicillin and other drugs, and radiographic material (renografin).

355
Q

Other abnormal structures in urine

A

tumor cells, viral inclusion cells, platelets, bacteria, fungi, parasites, and contaminants and artifacts.

356
Q

Phenylketonuria

A

sign of phenylalanine hydroxylase deficiency.

357
Q

Alkaptonuria

A

sign of hemogentisic acid oxidase deficiency

358
Q

Tyrosinuria

A

sign of multiple disorders of tyrosine metabolism

359
Q

Maple syrup urine disease

A

multiple disorders that prevent the conversion of keto acids to fatty acids.

360
Q

Homocystinuria

A

cystathione beta-synthase deficiency.